You are on page 1of 79

1-Maternal disease producing antibodies, which cross the placenta and cause

disease in the neonate include:

Idiopathic thrombocytopaenic purpura


Thyrotoxicosis
Diabetes mellitus
Myasthenia gravis
Huntington’s Chorea

Maternal antibodies may cross the placenta as early as the first trimester. Anti-
cardiolipin antibodies as seen in SLE may cause heart block. Anti-D antibodies
may cause erythroblastosis fetalis. Neonatal thyrotoxicosis may be due to maternal
anti-thyroid antibodies where the mother has Graves disease or Hashimoto’s
thyroiditis. Dystrophia myotonica may manifest in infants of affected mothers but
the mechanism is likely to be genetic. Maternal antibodies to HIV do cross the
placenta but only the virus, and not antibodies to it, cause the disease. Although
maternal diabetes can cause problems in the neonate, this is due to the high
concentration of glucose in the maternal blood crossing the placenta, not the anti-
bodies. Anti-acetylcholine receptor antibodies transferred across the placenta may
cause transient myasthaenia gravis which lasts 4-6 weeks in about 15% of neonates
born to affected mothers. Maternal autoantibodies seen in ITP may cross the
placenta and lead to destruction of fetal platelets resulting in moderate to severe
thrombocytopaenia. T T F T F

2-Regarding HiB vaccination, which of the following are true?

It is a live vaccine
It is contra-indicated in a child with a history of convulsions
It is given with the MMR immunisation
Boosters are given at monthly intervals for 3 months in children under 1 year old
It must not be given as a single injection with DPT

DTaP/IPV/Hib is a combined vaccine which contains the following active


ingredients: purified high dose diphtheria toxoid; purified tetanus toxoid; five
purified components of the Bordetella pertussis bacteria; three strains of
inactivated polio virus and a purified component of Haemophilus influenzae type b
bacteria attached to a tetanus toxoid carrier protein. It is usually given to babies at

1
the age of two, three and four months but check the NHS immunisation schedule
for the most recent details: http://www.immunisation.nhs.uk/ F F F T F

3-Which of the following are true regarding BCG vaccination?

Efficacy is demonstrated by a hypersensitivity response to vaccine administration


It is contraindicated in neonates
It offers protection against infection with Mycobacterium bovis as well as
Mycobacterium tuberculosis
It should be injected subcutaneously
It contains a live attenuated virus

Bacillus of Calmette and Gurin (BCG) is a vaccine against tuberculosis that is


prepared from a strain of the attenuated (weakened) live bovine tuberculosis
bacillus, Mycobacterium bovis (a bacteriumm, not a virus), that has lost its
virulence in humans through serial propagation in artificial media. The bacilli
retain sufficient immunogenicity to behave as a reasonably effective vaccine for
the prevention of human tuberculosis, particularly in the prevention of TB
meningitis and disseminated disease in early childhood. Protection against
pulmonary disease in later life is less certain. BCG vaccine should always be given
intradermally as subcutaneous administration is likely to be associated with
adverse effects including local abscess formation and suppurative lymphadenitis.
The Mantoux test results in a cutaneous hypersensitivity response to administered
PPD (purified protein dericative), but neither a cutaneous response to the vaccine
or to PPD are sufficient or required to demonstrate vaccine efficacy. F F T F F

4-In order to ensure meticulous care in hygiene, which measure is NOT


acceptable when preparing bottle feeds?

Using boiled water


Allowing the liquid to cool
Placing in the refrigerator
Using sterilised bottles
Warming the milk twice

Babies are at risk of infection due to their relatively poor immunity and by
warming the milk twice, you are allowing bacteria a chance to culture in the milk.5

2
5-MMR contraindications include which of the following?

Child treated with 2mg/kg/day prednisolone


Children with history of neonatal seizures
Previous vomiting after egg ingestion
History of febrile convulsions
Severe eczema

Contraindications include: acute fever, recent immunoglobulin administration,


administration of another live vaccine in the past 3 weeks, immunosuppression,
allergy to neomycin or kanamycin, pregnancy. Steroids, such as prednisolone,
given to children either orally or rectally at a daily dose of 2mg/kg/day for at least
one week or 1mg/kg/day for one month should be deemed as a contraindication
and administration on live vaccines should be postponed for at least 3 months after
immunosuppressive treatment has stopped.

In children with a significant history of an anaphylactic reaction to eggs, or who


have had egg allergy and chronic severe asthma the vaccination should be
administered in hospital. The vaccine is not however contraindicated in such
patients. T F F F F

6-Which of the following are true regarding Oral Polio Vaccine (OPV)?

It contains 3 strains of live attenuated virus


It is contraindicated in the presence of diarrhoea
It is contraindicated in children on oral steroids 2mg/kg/day
Use of the inactive vaccine reduces the carriage of the wild virus
Incidence of vaccine associated poliomyelitis is negligible

Oral polio ('Sabin') vaccine is given by mouth in contrast to the inactivated ('Salk')
vaccine (IPV) which is given IM. Polio vaccines are usually given at two months,
three months and four months of age, with a booster before school, usually
between 3 and 5 years of age and again, before leaving school, between 15 and 19
years old. Boosters thereafter are not normally necessary, unless travelling to an
area where polio is common, or likely to be exposed to people with polio.

3
The oral vaccine contains live virus particles which have been attenuated to reduce
the risk of neurological disease. The risk of vaccine associated poliomyelitis is
small but not negligible and this together with the elimination of wild polio from
the European Region was the principle reason for the change to the routine use of
IPV in the UK in 2006.

Three types of poliomyelitis virus (Types 1, 2 and 3) are included in the vaccine.

Contraindications:

 Postpone if acute illness with pyrexia, diarrhoea/vomiting


 Immunodeficiency/treatment with high doses of steroids/immunosuppresants
 First four months of pregnancy

TTTTF

7-Which of the following children should be given BCG immunisation?

Normal healthy infants with Asian mothers going to Bangladesh


Migrant from Africa
A 13-year-old child who is moving to a high risk area but who has had
immunisation as an infant
Child who has a contact with active pulmonary TB
Infant with a grandparent born in a high risk area

BCG is recommended for the following groups if BCG immunisation, as


evidenced by a characteristic scar, has not previously been carried out and they are
negative for tuberculoprotein hypersensitivity:

 all infants living in areas where the incidence of tuberculosis is greater than
40 per 100 000;
 infants with a parent or grandparent born in a country with an incidence of
tuberculosis greater than 40 per 100 000;
 previously unvaccinated new immigrants from countries with a high
incidence of tuberculosis;
 contacts of those with active respiratory tuberculosis;
 health service staff
 veterinary staff

4
 staff working in prisons, in residential homes and in hostels for refugees and
the homeless;
 those intending to stay for more than 1 month in countries with a high
incidence of tuberculosis
 neonates, infants, children or adults where immunisation is requested.
TTFTT

8-Which of the following are true regarding the Meningococcal vaccine which
is used in the UK immunisation schedule?

It protects against types B and C


It can cause meningoencephalitis
It is contraindicated in HIV infection
3 injections are given from 6 months of age
It is a live attenuated vaccine

Almost all childhood meningococcal disease in the UK is caused by Neisseria


meningitidis serogroups B and C. Meningococcal Group C conjugate vaccine
protects only against infection by serogroup C; it can be given from 2 months of
age. After early adulthood the risk of meningococcal disease declines, and
immunisation is not generally recommended after the age of 25 years.

Meningococcal Group C conjugate vaccine provides long-term protection against


infection. The recommended schedule consists of 3 doses given at 3, 4 and 12
months.

Side-effects of meningococcal Group C conjugate vaccine include redness,


swelling, and pain at the site of the injection, mild fever, irritability, drowsiness,
dizziness, nausea, vomiting, diarrhoea, headache, myalgia, rash, urticaria, pruritus,
malaise, lymphadenopathy, hypotonia, paraesthesia, hypoaesthesia, and syncope.
Hypersensitivity reactions and seizures have been reported rarely. Symptoms of
meningism have also been reported rarely, but there is no evidence that the vaccine
causes meningococcal C meningitis. There have been very rare reports of Stevens-
Johnson syndrome. The CSM has advised that vaccination provides benefit in
terms of lives saved and disabilities prevented.

Currently meningococcal vaccine is a purified heat stable extract from the


polysaccaride outer capsule of Neisseria meningitidis. 4F

5
9-Which of the following are true regarding vaccines?

Diptheria vaccine is a toxoid


Pertussis vaccine is a killed bacillus
Injected polio vaccine is a live attenuated virus
BCG is a live attenuated bacillus
Measles is a live attenuated virus

Previously virulent micro-organisms that have been inactivated (with chemicals or


heat) include vaccines against influenza, cholera, plague, and hepatitis A. Most
such vaccines may have incomplete or short-lived immune responses and are likely
to require booster shots.

Live, attenuated vaccines typically provoke more durable immunological


responses. Examples include yellow fever, measles, rubella, and mumps. BCG is a
live tuberculosis strain. The oral polio vaccine is a live attenuated virus but
injected polio vaccine is inactivated.

Inactivated toxic compounds (Toxoids) from micro-organisms include tetanus and


diphtheria. The pertussis vaccine is acellular and made from part of the pertussis
cell. T F F T T

10-Which of the following are true regarding deranged cellular immunity?

It often presents with severe viral infections


It is associated with Di George syndrome
It is associated with gram-positive severe infections
It is associated with prematurity
It occurs in agammaglobulinaemia

Cell mediated immunity is mediated by T cells and is involved in control of viral


infections. Abnormalities of cell mediated immunity occur in Di George syndrome
because of the absence of T cell maturation secondary to thymus abnormalities.
Protection from bacterial infection is classically associated with B cell
abnormalities and the consequent inability to produce antibodies. Prematurity is
associated with immaturity of the immune system but this mainly manifests as

6
abnormal antibody production. X-linked agammaglobulinaemia is a pure B cell
defect and affected children have normal cell mediated immunity.

TTFFF

11-Which of the following regarding the Fc component of antibody are true?

Determines antigen binding specificity


Determines the ability to cross the placenta
Determines the ability to bind with mast cells
When produced in excess, excreted in urine as Bence-Jones protein
Determines metabolic half life of the whole molecule

The basic unit of an antibody is a monomer. An antibody can be monomeric,


dimeric, trimeric, tetrameric, pentameric, etc. The monomer is a Y-shaped
molecule that consists of two identical heavy chains and two identical light chains
connected by disulphide bonds.

Each half of the forked end of the Y-shaped monomer is called the Fab fragment. It
is composed of one constant and one variable domain of each the heavy and the
light chain, which together shape the antigen binding site. The Fc fragment is the
stem of the Y and is composed from two heavy chains. It binds to various cell
receptors and complement proteins. In this way it mediates different physiological
effects of antibodies (opsonisation, cell lysis, mast cell, basophil and eosinophil
degranulation and other processes).

Bence-Jones proteins are free immunoglobulin light chains. F T T F T

12-The advantages of breast feeding are:

Reduces the risk of bacterial contamination


Protection of infection via passage of maternal antibody (IgE)
Promoted colonisation with lactobacilli
Improves maternal bonding
Cheaper

There are many economical, social and medical advantages of breast feeding.
Maternal antibodies in the breast milk are IgA not IgE. T F T T T
7
13-Which of the following is NOT a contra-indication to MMR vaccination:

Children receiving high-dose cortico-steroids


Children with an anaphylactic allergy to egg
Children with an allergy to neomycin
Children with an acute febrile illness
Children who have received an immunoglobulin injection within the last 3 months

All of the above are contra-indications as written in the BNF except egg allergy -
this is a common misconception. In children with a significant history of an
anaphylactic reaction to eggs, or who have had egg allergy and chronic severe
asthma, the vaccination should be administered in hospital. The vaccine is not
however contraindicated in such patients.

Contraindications include: acute fever, recent immunoglobulin administration,


administration of another live vaccine in the past 3 weeks, immunosuppression,
allergy to neomycin or kanamycin, pregnancy. Steroids, such as prednisolone,
given to children either orally or rectally at a daily dose of 2mg/kg/day for at least
one week or 1mg/kg/day for one month should be deemed as a contraindication
and administration on live vaccines should be postponed for at least 3 months after
immunosuppressive treatment has stopped. 2

14-Which vaccination is given by the intradermal route usually?

Inactivated Polio
Meningococcal C
MMR
DTP
BCG

BCG is the only vaccine given intradermally. The rest are given by the
intramuscular or deep subcutaneous route. 5

8
15-Which statements concerning the MMR vaccination are correct?

Previous measles is a contraindication


The first dose is given at 6 months
A booster dose is given at 3-5 years
Parotid swelling is a recognised side effect
Idiopathic thrombocytopenic purpura is a rare complication

Any child who has had measles, mumps or rubella should be given the MMR
vaccination regardless of previous infection (BNF). The first dose is given at 12-
15 months with a pre-school booster. ITP is a rare complication and the risk of
developing ITP is much less with MMR than with the actual diseases of measles,
mumps and rubella. Parotid swelling is also a recognised side effect. F F T T T

16-Which vaccination is NOT included in the 2, 3, 4 month ages of a child


according to the UK immunisation schedule?

Polio
Hib
DTP
Meningococcal C
BCG

BCG is usually given at birth in high risk neonates or at ages 10-14. 5

17-Regarding a squint in a child, which statement is correct?

The child does not require further investigation if it has been present for over
6 months
May be regarded as within acceptable limits if transient and only lasting a
few seconds up to the age of 18 months
Is more frequent in children who have not been vaccinated
Is more frequent in children with myopia than hypermetropia
In a younger child is more likely to be divergent rather than convergent

9
It is important to refer children with squints over the age of 6 months for further
investigation and prevent permanent blindness in that eye. Transient squints of
only a few seconds duration may be acceptable up to 18 months of age.
Hypermetropia is a common cause of squints in children. Convergent squints are
more common. 2

18-Consider the options presented below. Which screening does NOT occur in
the United Kingdom?

Heel Prick Test to screen for congenital hypothyroidism


Red Reflex to screen for congenital cataract
Echocardiogram in children at 6 weeks to screen for / diagnose cardiac
structural disease
Otoacoustic emission to screen for hearing impairment
Heel prick test to screen for phenylketonuria

Auscultation is used to pick up murmurs in children, not routine echocardiograms


at 6 weeks. The Guthrie test (heel prick test/blood spot screening test) in the UK
screens for phenylketonuria, congenital hypothyroidism, sickle cell disorders,
cystic fibrosis and Medium Chain Acyl CoA Dehydrogenase Deficiency
(MCADD). 3

19-A Primary Prevention

B Secondary Prevention C Tertiary Prevention

D Health Promotion E Health Education

F Child Health Surveillance G Public health

The definitions below are best described by which of the options above? Each
option may be used once, more than once, or not at all.

It is important to consider the child in society and the importance of looking


at child health in the community as a whole. Secondary prevention is
reduction of prevalence of disease by shortening duration or diminishing the
impact through early detection and prompt intervention (eg screening).
Health promotion is any measure that improves health or prevents disease.

10
Public health is a science preventing disease, prolonging life and promoting
health through the organised efforts of society.

Scenario 1 Reduction in number of new cases of a disease, disorder or condition


(accidents)

Primary Prevention Correct answer

Scenario 2 Any activity which promotes health through learning

Health Education Correct answer

Scenario 3 Reduction of impairment and disability, and minimising suffering (e.g.


multidisciplinary approach of a child with Down syndrome)

Tertiary Prevention Correct answer

It is important to consider the child in society and the importance of looking at


child health in the community as a whole. Primary prevention is reducing number
of cases of disease, disorder, conditions or accident from the very onset. Secondary
prevention is reduction of prevalence of disease by shortening duration or
diminishing the impact through early detection and prompt intervention (e.g.
screening). Tertiary prevention is the reduction of impairment and disability in a
child with a pre-existing illness or condition. Health promotion is any measure that
improves health or prevents disease. Public health is a science preventing disease,
prolonging life and promoting health through the organised efforts of society.

20-A Polio B Diptheria

C Tetanus D Pertussis

E Measles F Mumps

G Rubella H Haemophilus influenzae type B

It is important to recognise the symptoms of these serious diseases especially if


a child has not been vaccinated against them.

11
Scenario 1 A child develops fever and parotid gland enlargement

Mumps Correct answer

Scenario 2 A child has prolonged URTI with paroxysms of coughing, followed


by a whoop with vomiting. Diagnosis is confirmed by a nasopharyngeal
culture.

Pertussis Correct answer

Scenario 3 A child has a throat infection with pharyngeal exudates. There is


membrane formation and upper airway obstruction.

Diptheria Correct answer

Scenario 4 A child develops mild fever and is then left with residual paralysis.

Polio Correct answer

Scenario 5 A child has a maculopapular rash, fever, coryza, cough and


conjunctivitis

Measles Correct answer

These conditions are not common due to the widespread uptake of immunisations
in the UK, however it is important to recognise the symptoms of these serious
diseases especially if a child has not been vaccinated against them.

21-Which of the following are true regarding CD4 Lymphocytes?

Anaphylaxis is due to fixing antibodies


They are involved in delayed type hypersensitivity
They secrete IL-2 in response to antigen stimulus
They are the predominant intra-epithelial lymphocytes in the gut
They are processed in the thymus

12
CD4+ve lymphocytes are known as helper T cells and interact with MHC Class II
molecules whereas CD8+ve T cells interact with MHC Class I molecules. CD4+ve
cells are involved in antigen specific responses as well as delayed type
hypersensitivity and secrete a number of cytokines including IL-2 in response to
antigen stimulation. Intra-epithelial lymphocytes in the gut are dominated by T
cells bearing the gamma/delta receptor and are not CD4+ve. All T cells are
processed in the thymus where self-reactive cells are deleted. F T T F T

22-Which of the following are true, regarding antibodies?

Hereditary angio-oedema is associated with C1 esterase inhibitor deficiency


The activity of hereditary angio-oedema is exacerbated by the menstrual cycle
Urticaria may have a non-allergic cause
Anaphylaxis is due to fixing antibodies
Serum sickness is due to antibodies against antigens found on RBC during a blood
transfusion

Hereditary angio-oedema is due to the absence or low level of C1 esterase inhibitor


(C1-INH), although it may also be caused by the abnormal function of C1-INH in
the presence of normal levels of the protein. Stress such as trauma or surgery may
precipitate attacks, which may also be exacerbated by changes in the menstrual
cycle. Urticaria occurs as a result of IgE interaction with basophils leading to the
release of mediators, which result in mucosal oedema. While it is thus usually due
to allergy, the trigger for an urticarial response may not always be found.
Anaphylaxis is due to IgE antibodies. Serum sickness is due to antibodies forming
to foreign proteins such as anti-sera to snake bites and occurs 7-10 days after
injection of the foreign protein. Antibodies to red cells cause Type II
hypersensitivity reactions (also known as cytotoxic reactions). T T T F F

23-Only IgE-mediated allergic reactions can be formally tested by skinprick


testing. Adverse reactions to which of the following substances can be tested in
this manner?

Morphine
Radiocontrast media
Scombrotoxins
Colloid plasma expanders

13
Latex

Latex can induce allergy through IgE bound to mast cells.

All the others induce histamine release via their direct effects on mast cells, except
for scombroid fish poisoning, which is related to the heat-stable toxin in tuna,
mackerel, mahi mahi, etc and causes immediate diffuse redness, diarrhoea and
vomiting. 5

24-Which of the following adverse food reactions is mediated by IgE-


dependent mechanisms and hence can be ascertained by skinprick testing?

Monosodium glutamate in Chinese food


Scombroid fish poisoning
Sulphites on prepacked salads
Salicylate-induced urticaria
Kiwi fruit
Kiwi fruit is a member of the latex-associated foods and adverse reactions to this
fruit are mediated by IgE. All the others are examples of food intolerance, so that
detailed history-taking is essential to making the correct diagnosis. Scombroid fish
poisoning causes immediate diffuse redness, diarrhoea and vomiting following the
consumption of fish such as tuna, mackerel and mahi mahi. Monosodium
glutamate causes abdominal bloating and vomiting – the so-called ‘Chinese
restaurant syndrome’. Sulphites on prepacked salads causing asthma is called
the ‘Salad-bar syndrome’. 5

25-In a patient with anaphylaxis, which of the following should be given to


inhibit the important late-phase reaction?

Antihistamines
Epinephrine
Leukotriene inhibitor
Hydrocortisone
NSAID

14
Hydrocortisone blocks the generation of leukotrienes and prostaglandins, and
hence prevents the late-phase reaction often characterised by asthma. It should be
given intravenously/intramuscularly at a dose of 100–200 mg.

None of the other agents listed above affect this aspect of anaphylaxis. Indeed
leukotriene inhibitors and NSAIDs are not part of the emergency management of
acute anaphylaxis.

Approximately 30% of deaths related to anaphylaxis occur as a consequence of


this late-phase reaction. 4

26-In an 18-year-old, recurrent and/or severe infections with which of the


following organisms would make you suspect immunodeficiency associated
with hypogammaglobulinaemia?

Haemophilus influenzae
Streptococcus pyogenes
Herpes simplex virus
Candida spp
Pneumocystis jiroveci

Infections with Haemophilus influenzae and Streptococcus pneumoniae occur with


this condition, not Streptococcus pyogenes. Pneumocystis jiroveci (formerly P.
carinii), viral and candidal infections are associated with T-cell deficiencies,
including HIV infection. 1

27-Immunological investigations in a patient with renal disease are important


in the diagnostic work-up, which of the following statements is correct?

Henoch–Schönlein purpura is associated with IgG in the mesangium


SLE is typically associated with sparse deposits of IgG and complement in
the glomeruli
C3 nephritic factor is associated with mesangiocapillary glomerulonephritis
type I
Minimal-change glomerulonephritis is associated with
hypocomplementaemia
Antiglomerular basement-membrane antibodies are associated with
Goodpasture’s disease

15
Minimal-change glomerulonephritis is not associated with complement activation.
C3 nephritic factor is associated with mesangiocapillary glomerulonephritis type 2,
not type 1. SLE nephritis is an immune complex problem; hence deposits of IgG
and complement are prolific.

Henoch–Schonlein purpura is typically associated with IgA in the mesangium,


hence immunological examination of both a blood and renal biopsy will help to
define the underlying process. 5

28-Anti-Ro antibodies in isolation (negative ANA) occur in which of the


following conditions?

Sjögren’s syndrome
Sicca syndrome
Systemic lupus erythematosus (SLE)
Scleroderma
Polymyositis
ANA-negative lupus is uncommon. ANA-negative SLE patients are usually Ro-
positive and have skin rashes with or without photosensitivity. In Sjögren’s
syndrome, Ro and La antibodies commonly coexist. Sicca syndrome is not
typically associated with autoantibodies. 3

29-Haemolytic disease of the newborn is typically restricted to the presence of


Rhesus antigens on red cells rather than ABO antigens. Predominantly, such
anti-Rh antibodies cross the placenta during the third trimester.

Which of the following statements best explains the background physiology?

Antibodies to ABO blood groups are IgM, whereas antibodies to Rhesus


antigens are IgG
Antibodies to ABO blood groups are IgG, whereas antibodies to Rhesus antigens
are IgM
Antibodies to ABO blood groups are IgA, whereas antibodies to Rhesus antigens
are IgG
Antibodies to Rhesus antigens are IgD, whereas anti-ABO blood groups are IgM

Antibodies to Rhesus antigens are IgE, whereas anti-ABO blood groups are IgG

16
IgG antibodies to Rhesus antigens can cross the placenta during the last trimester,
whereas ABO antibodies are IgM and hence cannot cross the placenta. The
function of serum IgD is unknown. The transplacental passage of immunoglobulin
only applies to IgG. 1

30-Which of the following are true regarding T-cells?

60-80% of total white cell count


They are more frequent than B cells
They mature in the thymus
They are associated with memory responses
Defects can occur in Di George Syndrome

Normal adult levels of white blood cells are 4,500-11,000 cells per microlitre of
blood. Lymphocytes account for approximately 25-45% of the total white blood
cell count; the normal range is 1,000-4,800 lymphocytes per microlitre of blood in
adults and older children (> 5 years of age), with higher circulating numbers in
infants and young children. Of the total lymphocytes, 60-80% are T cells and
approximately 15% are B cells. Mature functional T cells undergo differentiation
and maturation in the thymus. F T T T T

31-In skin prick testing for sensitivity to respi ratory allergens, the cutaneous
reaction is influenced by which of the following?

Recent use of inhaled salbutamol


Regular inhaled steroids
Cigarette smoking
Severe eczema
Oral antihistamine use

The diagnosis of allergic disorders is dependent on a careful history exploring the


relationship between symptom onset and exposure to potential allergens.
Determination of specific IgE (either through serum antibody, by RAST or skin-
prick tests) is useful in either confirming a clinical impression of allergy, or
sometimes in identifying a particular culprit from a small number of carefully-
selected potential allergens.

17
Skin-prick tests when carried out correctly provide a quick, visual, semi-
quantitative determination of the presence of specific IgE (e.g. to house dust mite
or to peanut) present on the surface of mast cells in the skin. A positive result is
indicated by a wheal (± erythema) at the test site 10-15 minutes after placing
prick tests for the allergen and a positive (histamine) control. The test is relatively
non-invasive and parents and children often appreciate the benefit of seeing (if not
scratching) the result, without the need to return to clinic at a later date for results.
It is important that negative and positive controls are tested together with potential
allergens. Testing may be difficult in children with extensive skin disease (eczema)
and false positives are common in highly atopic individuals, leading to real
difficulties in test interpretation. For this and other reasons, blind testing of a wide
range of allergens in the search for a hidden culprit is rarely helpful and not
recommended.

False negatives are likely in children who have taken recent antihistamine
preparations (they should be avoided 48 hours prior to the test) and to a lesser
extent in those recently exposed to systemic or topical (but not inhaled) steroids.

Skin-prick testing is often useful in confirming a clinical diagnosis of food allergy


where diagnosis can be followed by dietary exclusion. The issues are more
complex with aero-allergens (e.g. for house-hold pets or outdoor pollen) as
avoidance is usually more difficult. F F F T T

32-Which of the following are true regarding complement?

IgA activates complement via the classical pathway


Severe burns activate complement
The alternative pathway is activated when complement levels in the classical
pathway run out
C2 is involved in both the classical and alternative pathways
C5 is increased in meningococcal meningitis

The complement cascade consists of 3 separate pathways that converge in a final


common pathway. The pathways include the classic pathway (C1qrs, C2, C4), the
alternative pathway (C3, factor B, properdin), and the mannan-binding pathway
(mannan-binding lectin [MBL]). These 3 pathways converge at the component C3.

The classical pathway is activated by immune complexes, whereas the alternative


pathway is activated by lipopolysaccharide component of the cell wall of gram

18
negative bacteria. Both result in conversion of C3 to C3b, which activates the lytic
complement sequence. The terminal complement pathway consists of all proteins
activated after C3; the most notable of these is the C5-C9 group of proteins known
collectively as the membrane attack complex (MAC), of which C5 has been
implicated in meningococcal meningitis. F T F F T

33-Which of the following are true with regard to IgE?

It can cross the placenta


Serum concentration is equal to that of IgG
It is acutely increased in an acute asthma attack
It is associated with type II hypersensitivity
It is associated with mast cell function

The antigen-specific IgE interacts with mast cells and eosinophils to protect the
host against the invading parasite. The same antibody-cell combination is also
responsible for typical allergy or immediate (Type I) hypersensitivity reactions
such as hayfever, asthma, hives and anaphylaxis. F F T F T

34-Vasculitis occurs in which of the following conditions?

Systemic lupus erythematosus


Immune thrombocytopenic purpura
Necrotising enterocolitis
Stevens – Johnson syndrome
Kawasaki disease

Vasculitides are characterised by inflammatory destruction of blood vessels,


occurring in veins (phlebitis) or arteries (arteritis). Some conditions have vasulitis
as their main feature and include:

 HSP
 Kawasaki Disease
 Polyarteritis nodosa
 Wegener’s granulomatosis
 Takayasu’s arteritis
 Giant cell arteritis (temporal arteritis)

19
In some conditions vasculitis may be a symtpom and include:

 SLE
 Lymphomas
 Infections
 In response to some drugs as part of Stevens-Johnson syndrome

Purpura in ITP results from thrombocytopenia. There is no exact cause for


necrotising colitis known, but it is not thought to be due to a vasculitis. T F F T T

35-Which of the following are absolute contra-indications to Pertussis


immunisation?

Epilepsy
Family history of convulsions after immunisation
Eczema
Autism
Previous severe local reaction to immunisation

Since 2006 acellular pertussis vaccines have been used for routine immunisation in
the UK and this change, together with an assessment of vaccine reactions has led to
a significant change in the contraindications and warnings relating to pertussis
immunisation.

The only remaining absolute contraindications for pertussis-containing vaccines


are:

 Confirmed anaphylactic reaction to a previous dose of pertussis-containing


vaccine, or
 Confirmed anaphylactic reaction to neomycin, streptomycin or polymyxin B
(which may be present in trace amounts in the vaccine).

Of particular note (and in contrast to previous advice), immunisation with


pertussis-containing vaccines should not be withheld despite a previous history of
convulsions, high fever, hypotonic-hyporesponsive episodes (HHE), persistent
crying, screaming or severe local reaction following immunisation.

By the time of school entry children in the UK should have received 4 doses of
pertussis-containing vaccines (DTaP/Hib/IPV at ages 2, 3 & 4 months, and DTaP

20
or dTaP at age 3.5 to 4 years (a preschool booster). The booster dose at school
entry was added in 2006 at the same time that the change was made from whole-
cell to acellular pertussis component vaccines. 4F

36-The following immunoglobulin are responsible for:

IgA leads to bacterial agglutination


IgD is involved in inhibition of viral replications
IgE : Type 1 sensitivity reaction
IgG : neutralisation of bacterial toxins
IgM : complement activation

Immunoglobulins are antibodies. They are made of gamma globulins and in


humans there are five types.

1. IgA is found in mucosal areas, such as the gut and respiratory tract. It is also
found in saliva, tears and breast milk. It prevents bacterial infection y
leading to agglutination.
2. IgD upregulates cellular response to viral infections by activating basophils
and mast cells.
3. IgE binds to allergens and triggers histamine. It is associated with a Type I
hypersensitivity reaction.
4. IgG is split into 4 further subclasses. It provides the majority of antibody
based immunity. It is the only antibody that crosses the placenta.
5. IgM leads to complement activation. T F T T T

37-Which of the following mediators attract cells in inflammation?

Interleukin 4
Interleukin 8
Leukotrine B4
C3b
C5a

21
IL-4: involved in proliferation of B cells, and the development of T cells and mast
cells. Important role in allergic responses.

IL-8: Neutrophil chemotaxis

C3b acts as an opsonizer.

C5a is an anaphylatoxin, causing the release of histamine from mast cells. It is also
an effective leucocyte chemoattractants, causing the accumulation of white blood
cells, especially neutrophil granulocytes, at sites of complement activation.

Leukotrienes are important agents in the inflammatory response. Some such as


LTB4 have a chemotactic effect on migrating neutrophils, and as such help to
bring the necessary cells to the tissue. Leukotrienes also have a powerful effect in
vasoconstriction particularly of venules and of bronchoconstriction, they also
increase vascular permeability. F T T F T

38-Which of the following are true statements regarding common variable


immune deficiency?

Patients are particularly prone to viral infections


Circulating B cell numbers are usually normal
IgG levels are normal
It typically manifests autosomal recessive inheritance
Patients are at an increased risk of malignancy

Common variable immunodeficiency (CVID) is a disorder that involves the


following: (1) low levels of most or all of the immunoglobulin (Ig) classes, (2) a
lack of B lymphocytes or plasma cells that are capable of producing antibodies,
and (3) frequent bacterial infections.

CVID is a common immune disorder and is, in fact, the most prevalent primary
immunodeficiency. It can be inherited in autosomal dominant or recessive manner.
However, it is diverse, both in its clinical presentation and in the types of
deficiency. Although decreased serum levels of immunoglobulin G (IgG) and
immunoglobulin A (IgA) are characteristic, approximately 50% of patients with
the deficiency also have diminished serum immunoglobulin M (IgM) levels and T-
lymphocyte dysfunction. The primary cause of CVID remains unknown. In
patients with CVID, the risk of certain malignancies is high. Lymphomas of a B-

22
cell phenotype are of particular concern and malignancy is most likely associated
with the Epstein-Barr virus. F T F F T

39-Which of the following antibodies are correctly linked with a disease with
which they are associated?

Antiesterase Ab in myasthaenia
Antiendomysial Ab and coeliac disease
Anti-acetylcholine antibodies and spinal muscular atrophy
21 hydroxylase Ab and congenital adrenal hyperplasia
Antiepidermal Ab and vitiligo

Autoantibodies may be present in patients with myasthenia (antibodies against the


acetylcholine receptor) and vitiligo but do not match those given in the question.
FTFFF

40-Which of the following is/are true regarding overwhelming septicaemia in


post-splenectomy patients?

It can be fully prevented with vaccination


It usually involves gram-negative organisms
It is only a significant problem for two/three years post splenectomy
It is due to hypogammaglobulinaemia
Immunisation after removal of the spleen has a better outcome

There is an increased risk of pyogenic infections, including pneumonia, septicemia


and meningitis caused by pneumococcus and other polysaccharide encapsulated
organisms (e.g. Haemophilus influenzae type b) after splenectomy.

The mechanism for this immune deficiency is not entirely clear - it would seem to
be multifactorial. Immunoglobulin levels generally remain normal. As
pneumococcal infection is numerically the greatest risk, prophylaxis with penicillin
is used. However, particularly in young children, awareness of the risks from other
infections should be maintained.

23
Vaccination with pneumococcal and Haemophilus influenzae type B vaccines may
diminish the risk but do not remove the need to give penicillin. Immunisation
should be performed prior to elective splenectomy. 4F

41-In a 16-year-old female with systemic lupus erythematosus, which of the


following investigations/statements is true?

CRP is typically normal in non-infected patients with active disease


Rheumatoid factor is positive in < 5% of patients
Neutropenia is more common than lymphopenia
The low-dose oral contraceptive pillis contraindicated
Pulmonary fibrosis is a common disease complication

The CRP is often normal in SLE even if the patient has active disease. An elevated
CRP therefore may indicate the presence of infection.

Rheumatoid factor tests can be positive in up to 40% of affected individuals.


Lymphopenia is more common than neutropenia. The oral contraceptive pill is not
contraindicated and pulmonary fibrosis is an unusual late complication. 1

42-Which of the following arthritides is frequently associated with the eye


inflammation described?

Juvenile idiopathic arthritis and uveitis


Seronegative arthropathies and retinal vasculitis
Systemic lupus erythematosus and pars planitis
Behçet’s disease and uveitis
Sarcoid and anterior uveitis

Behçet’s disease is most commonly associated with posterior uveitis,


although hypopyon can also be seen. The other typical pairings should be juvenile
idiopathic arthritis with scleritis, seronegative arthropathies with anterior uveitis,
SLE with retinal vasculitis and sarcoid with uveitis. 4

43-High titres of antithyroid microsomal and antithyroglobulin antibodies


would suggest which of the following diagnoses in a patient presenting with a
complaint of tiredness?

24
Hashimoto’s thyroiditis
Reidel’s thyroiditis
Graves disease
Hypoparathyroidism
Idiopathic hypothyroidism

This finding in Hashimoto’s thyroiditis is characteristic, but lower titres can


occur in Reidel’s thyroiditis and Graves disease. High titres of these antibodies
in euthyroid individuals indicate the possibility of future thyroid failure, but this
may be many years away; hence the need for thyroid function tests every 1–2
years in such individuals. 1

44-Which of the following statements in relation to skin disease is true?

Eye involvement is unusual in benign mucous membrane pemphigoid


The rash of herpes gestationis most commonly affects the chest
Pemphigus is associated with subepidermal bullae
Epidermolysis bullosa acquisita is strongly associated with HLA-DR4
Nasal swabs should be routinely checked in patients with recurrent
staphylococcal abscesses

Eye involvement is common in pemphigoid. Pemphigoid is associated with


subepidermal bullae, and in pemphigus they are intraepidermal. Epidermolysis
bullosa acquisita is associated with DR-2 and not DR-4. The rash of herpes
gestationis is typically found on the lower abdomen. 5

45-A 16-year-old suffers recurrent, proven bacterial chest infections requiring


regular antibiotic treatment.

Which of the following is the best way to exclude antibody deficiency?

Serum immunoglobulins
Immunoglobulin subclasses
Specific antibodies to haemophilus and pneumococci
Complement levels
Mannan-binding proteins

25
Normal specific antibodies to pneumococci, haemophilus and also tetanus will
exclude a significant antibody deficiency. Immunoglobulins and subclasses can be
normal even though there is a profound absence of specific antibodies. 3

46-A patient collapses during induction with a general anaesthetic.

Which of the following investigations will be most useful in subsequently


establishing an IgE-mediated process (anaphylactic mechanism)?

Elevated serum tryptase at approximately 1 hour after collapse


Total serum IgE level
Skin-prick tests to anaesthetic agents
Serum/plasma C3 and C4 levels
Specific IgE to latex

Although tryptase measurements indicate mast-cell degranulation, they do not


point to the triggering mechanism.

Total serum IgE is a test with little clinical value except in the interpretation of
specific IgE measurements.

Skin-prick tests performed at neat and 1:10 dilutions are the recognised
investigations for identifying which anaesthetic agents were problematic.

Plasma complement levels are rarely helpful.

Specific IgE to latex for investigating latex allergy may be helpful but is unlikely
to be the cause of a reaction at induction, i.e. before the surgeon has a gloved hand
inside the patient. 3

47-In a patient with suspected hereditary angio-oedema (HAE), which of the


following is most likely to be helpful in identifying this clinical condition?

Persistently low C3 levels, including between attacks


Persistently low C4 levels, including between attacks
Search for a paraprotein
Drug history of taking ACE inhibitors

26
Presence of urticaria with angio-oedematous swellings

A lack of C1 inhibitor occurs in approx 85% of cases of HAE and a functional


deficiency in the remaining 15%. C4 levels usually remain low even between
attacks, whereas C3 levels remain normal. The measurement of C4 is therefore
helpful in identifying those 15% of patients who have a normal amount of C1
inhibitor which functions poorly but gives normal levels when measured.
Paraproteins can occasionally bind C1 inhibitor to create a functional deficiency.
ACE inhibitors may exacerbate the angioedema in HAE and hence should be
avoided. Urticaria is never a feature of HAE. A drug history would be useful but
would not identify the HAE condition. 2

48-A teenager complains of an immediate intense itching in her throat when


eating apples, but says that she can eat cooked ones. She mentions that she has
allergic rhinitis but the current symptoms occur in April and May and not the
typical June/July period associated with grass pollen.

What is the most likely diagnosis?

Birch-associated oral allergy syndrome


Food intolerance
Allergy to the wax coating on apples
Latex allergy
Salicylate sensitivity

The birch pollen-induced oral allergy syndrome occurs with stoned fruits, apples,
carrots and potatoes. However, this only happens with the raw form as cooking
denatures the allergen. The birch-tree pollen season is usually in April/May, giving
the typical rhinitis symptoms. Actual throat swelling is unusual. Immediate
symptoms (minutes) are untypical of food intolerance and the wax coating on
apples is not a cause of allergies. Latex allergy can be associated with certain foods
such as bananas, avocado, kiwi and melon, but this allergen is heat-stable. Most
apples contain a considerable amount of salicylate, which can induce urticaria in
aspirin-sensitive individuals; however, this is not usually associated with
pharyngeal itching. 1

27
49-A teenager who has been doing work experience presents with red weals
and itchy hands within 20 minutes of wearing latex gloves.

Which of the following mechanisms is most likely to be relevant?

Contact dermatitis
Complement-mediated
Immune complex-mediated
Delayed-type hypersensitivity
IgE-mediated sensitivity

This is an immediate hypersensitivity (type-1) reaction and hence will be IgE-


mediated. Delayed-type (4) hypersensitivity, (e.g. contact dermatitis, Mantoux
test), has an onset that usually exceeds 24 hours. Complement-mediated (type-2
hypersensitivity) and immune complex (type-3 hypersensitivity) reactions typically
produce vascular damage and therefore bullae and petechiae would predominate in
the skin. 5

50-During the last trimester, IgG is actively transported across the placenta to
supply passive immunity to the foetus.

Which disease occurring during pregnancy is most likely to lead to the


neonate having low immunoglobulin levels and hence being prone to bacterial
infections?

Intestinal lymphangiectasia
Systemic lupus erythematosus
Myasthenia gravis
Ulcerative colitis
Prematurity

Typically, intestinal lymphangiectasia has its onset during pregnancy, and is


sometimes diagnosed when the neonate is investigated for bacterial infection.
Although systemic lupus erythematosus with severe nephrotic syndrome could also
produce hypogammaglobulinaemia, this would be a rare occurrence in a neonate.
Myasthenia gravis due to the transfer of maternal IgG antibodies across the
placenta can result in neonatal myasthenia. Significant protein loss from the gut in

28
ulcerative colitis is highly unusual, especially in a neonate. Prematurity does
reduce the time for placental transfer of IgG whilst the foetus is in utero and hence
produces a so-called ‘physiological hypogammaglobulinaemia’, but this is
not a disease. 1

51-You see a schoolboy who has a part-time job ‘washing up’ at a


restaurant. He presents with an eczematous rash on his hands suggestive of
contact dermatitis, possibly related to wearing latex gloves at work. You
discuss confirmatory tests.

Which of the following tests is most likely to be helpful establishing the


diagnosis?

Atopy patch testing


Skin-prick testing
Patch testing
Intradermal testing
Direct exposure tests to gloves

Patch testing is the classical method for investigating contact dermatitis (which is
T-lymphocyte mediated). Skin-prick testing investigates IgE-mediated reactions,
typically to aeroallergens and food. The atopy patch test is a research tool used to
investigate the possible role of food in exacerbations of eczema. Direct exposure to
gloves will simply support the history. Intradermal testing is usually reserved in the
UK for drug testing. 3

52-A 15-year-old girl with partial lipodystrophy presents with a 3- month


history of increasing swelling of her legs, which is now up to her knees.
Urinalysis shows heavy proteinuria but no haematuria. A diagnosis of
nephrotic syndrome is therefore made.

Which investigation is most likely to help in the definition of the underlying


renal pathology?

IgA enterocyte antibodies


Complement studies
Antineutrophil cytoplasmic antibodies (ANCA)
Antiglomerular basement membrane antibodies (anti-GBM)

29
Antistreptolysin titre (ASOT)

Nephrotic syndrome in an individual with partial lipodystrophy is likely to be


mesangiocapillary nephritis with C3 nephritic factor. The latter is an autoantibody,
and interacts with the complement pathway to give a low C3 with a normal C4
level. Serum immunoglobulins are likely to show reduced IgG and raised IgM
levels, which is typical of the nephrotic syndrome. Anti-GBM antibodies are likely
to be negative as the patient is young and there is no haematuria or pulmonary
involvement, which would be suggestive of Goodpasture’s syndrome. 2

53-A 16-year-old youth presents with lethargy and abdominal bloating. He


has lost 5 kg in weight over the last 6 months. Examination confirms a thin
teenager with obvious pallor.

Which is the most appropriate serological test to investigate possible


malabsorption?

Striated muscle
IgG antiendomysial antibodies
IgA antigliadin antibodies
IgA enterocyte antibodies
IgA antiendomysial antibodies

Small-bowel, duodenal biopsy obtained via an upper endoscopy is the ‘gold


standard’ investigation for coeliac disease. However, serological testing using
IgA antiendomysial antibodies is the most discriminating of the above choices. The
newer test, becoming increasingly available, is IgA antitransglutaminase. IgG
antiendomyseal antibodies will be of value in the diagnosis of coeliac disease in
IgA-deficient individuals, because in these individuals the IgA antiendomysial
antibody test will be negative, therefore it is vital to check IgA levels in patients
with suspected coeliac disease. 5

54-Antibodies to which one of the following are found most frequently in


patients with myasthenia gravis?

Acetylcholine esterase
Acetylcholine receptors
Myelin

30
Striated muscle
Tensilon

Myasthenia is characterised by fatigability of striated muscle. It is an autoimmune


disease with antibodies against acetylcholine receptors. There are at least four
subgroups of myasthenia which differ in age and sex spread, systemic
muscle/ocular muscle/both, thymic involvement, antibody titres, HLA associations
and treatment. Antibodies to acetylcholine esterase (the enzyme which breaks
down acetylcholine) would, in theory, increase the amount of available
acetylcholine. Myelin is not a primary target for autoantibodies in this disease.
Tensilon (edrophonium) has short-acting anticholinesterase activity and is used to
aid diagnosis of the disease. Electromyography (EMG) may be helpful in
diagnosis, and around 90% of patients have raised titres of anti-acetylcholine
receptor antibodies. 2

55-Which of the following is associated with hereditary angio-oedema?

Low levels of C1 inhibitor


High levels of C1 inhibitor
C3 deficiency
High levels of C4 complement during an attack
Deficiency of the membrane-attack complex (C5b–9)

This disorder is due to low levels of the C1 inhibitor of the complement system and
is one of the commonest complement deficiencies. Low levels of the C1 inhibitor
allow C1 to act on C4 and C2. This in turn produces kinin-like products that cause
the angio-oedema. Low levels of C4 are found during an attack. C3 levels are
normal. Membrane-attack complex deficiencies leave patients particularly
susceptible to neisserial infection. In a few cases C1 inhibitor levels are normal but
defective. The skin lesions are not itchy, unlike allergic urticaria. Painful intestinal
involvement can occur. Triggers include stress, infection and menstruation.
Danazol may be used in treatment. Acquired C1 inhibitor deficiency may be
associated with lymphoproliferative disease and infection. 1

31
56-Which of the following is associated with the correct disease?

HLA DR4 – ankylosing spondylitis


HLA B27 – Behçet’s disease
HLA B5 – haemochromatosis
HLA A3 – multiple sclerosis
HLA Cw6 – psoriasis

Regions of HLA are found on chromosome 6:

HLA-A Haemochromatosis (A3)


HLA-B Behçet’s syndrome (B5)
Ankylosing spondylitis (B27)
Reiter’s syndrome (B27)
HLA-C Psoriasis (Cw6)
HLA-DR Goodpasture’s syndrome (DR2)
Multiple sclerosis (DR2)
Addison’s disease(DR3)
Rheumatoid disease (DR4)

57-Which of the following statements concerning hyperacute rejection after


renal transplantation is correct?

Occurs at least three days after surgery


Is successfully treated with ciclosporin A
Is caused by ciclosporin A
Is mediated by preformed circulating antibodies
Is largely a B-cell-mediated response

Hyperacute rejection of a donor kidney may only take a few minutes once the
organ has been vascularised. Preformed circulating antibodies react with MHC
class 1 antigens on the transplanted kidney. A reaction ensues involving
complement molecules, an influx of polymorphs and the aggregation of platelets.

32
Blood vessels supplying the organ become obstructed causing ischaemia.
Ciclosporin A has no role in the treatment of hyperacute reactions: the only
treatment is removal of the organ. However, circulating antibodies mean the next
organ must be MHC 1-matched. Matching for MHC 2 is well known to prolong
graft survival. The use of ciclosporin has revolutionised donor organ survival. It
modifies T-cell responses, but is itself nephrotoxic. 4

58-Which of the following diseases is correctly matched to the


immunodeficiency?

Ataxia–telangiectasia – absent NBT (neutrophil nitroblue tetrazolium)


reduction
Bruton’s disease – impaired phagocytosis
Chronic granulomatous disease (CGD) – hypogammaglobulinaemia
Chédiak–Higashi – reduced IgA levels
DiGeorge syndrome – absent T-cell function

Ataxia–telangiectasia presents in childhood with cerebellar ataxia, impaired cell-


mediated immunity and productions of antibody. It is an autosomal-recessive
condition. Bruton’s disease is an X-linked hypogammaglobulinaemia due to
absent mature B cells. Recurrent pyogenic infections occur once maternal antibody
levels fall. CGD (chronic granulomatous disease) is caused by a failure of
intracellular killing (no respiratory burst). There are various types and screening
for CGD is by the nitroblue tetrazolium (NBT) test. Chédiak–Higashi is a
disorder affecting neutrophil chemotaxis and is inherited as an autosomal-
recessive. DiGeorge syndrome consists of hypoparathyroidism (hypocalcaemic
convulsions in newborns), cardiac anomalies, abnormal facies and absent T-cell
function (impaired cell-mediated immunity). In this condition the third and fourth
branchial arches, and therefore the thymus and parathyroid, fail to develop. As a
result, cardiac outflow tract anomalies occur (interrupted aortic arch, truncus
arteriosus and teratology of Fallot). 5

59-You are reviewing an HLA tissue typing result. The sample has been found
to be HLA B5-positive on typing.

Which of the following diseases is most closely associated with HLA B5?

Dermatitis herpetiformis

33
Behçet’s syndrome
Grave’s disease
Addison’s disease
Sjögren’s syndrome

The major histocompatibility complex (MHC) is a cluster of genes located on the


short arm of chromosome 6, and it codes for a series of molecules known as the
‘human leucocyte antigens’ (HLA). HLA molecules are distributed
throughout the body, and it is through differential HLA expression that cells are
classified as ‘self’ or ‘non-self’. Class-I molecules (subtypes A, B and
C) are expressed on all cell types except erythrocytes and trophoblasts. They
interact with CD8-positive T cells and are involved in driving cytotoxic reactions.

Behçet’s syndrome is associated with HLA B5 positivity, as is polycystic


kidney disease and ulcerative colitis. All the other stems mentioned in this question
are associated with the B8-DR3 subtype, as are autoimmune hepatitis, idiopathic
membranous glomerulonephritis, myasthenia gravis and systemic lupus
erythematosus. 2

60-A patient presents with eczema, thrombocytopenia and recurrent


infections. What is the most likely diagnosis?

Wiskott–Aldrich syndrome
Hyper-IgE syndrome
Gaucher’s disease
IgA deficiency
Malignancy

Wiskott-Aldrich syndrome is an X-linked disease, characterised by eczema,


thrombocytopenia and repeated infections and is caused by mutations in the WASP
gene. The WASP protein is expressed in cells of all haematopoietic lineages. It
may serve a cytoskeletal organising role for signalling elements that are
particularly important in platelets and T cells. The platelets are small and have a
shortened half-life. Affected male infants often present with bleeding and most do
not survive childhood, dying of complications of bleeding, infection or
lymphoreticular malignancy. The immunological defects include low serum
concentrations of IgM, while IgA and IgG are normal and IgE is frequently

34
increased. The number and class distribution of B lymphocytes are usually normal.
1

61-A patient presents with facial abnormalities that may include abnormal
ears, a shortened philtrum, micrognathia and hypertelorism.

Which cells is this patient lacking?

T lymphocytes
B lymphocytes
Erythrocytes
Melanocytes
Leucocytes

This classic example of isolated T-cell deficiency results from maldevelopment of


thymic epithelial elements derived from the third and fourth pharyngeal pouches.
The gene defect has been mapped to chromosome 22q11 in most patients with the
DiGeorge syndrome, and to chromosome 10p in others. Signs of defective
development of organs dependent on cells of embryonic neural crest origin
include: congenital cardiac defects, particularly those involving the great vessels;
hypocalcaemic tetany, due to failure of parathyroid development; and absence of a
normal thymus. Facial abnormalities may include abnormal ears, a shortened
philtrum, micrognathia and hypertelorism. Serum immunoglobulin concentrations
are frequently normal, but antibody responses, particularly of IgG and IgA
isotypes, are usually impaired. T-cell levels are reduced, whereas B-cell levels are
normal. Affected individuals usually have a small, histologically normal thymus
located near the base of the tongue or in the neck, allowing most patients to
develop functional T cells in numbers that may or may not be adequate for host
defence. 1

62-A 16-year-old girl presents to Accident & Emergency with an acute


hereditary angioedema involving the larynx.

What is the appropriate immediate management?

Antihistamines
IV hydrocortisione
Fresh frozen plasma

35
C1 inhibitor concentrate
Recombinant C1 inhibitor

This condition is due to deficiency of C1 inhibitor. This causes uncontrolled


activation of the classical complement pathway, resulting in angioedema. Patients
often present with orofacial swelling, which can cause respiratory obstruction and
abdominal pain. They do not usually get urticaria or pruritis (this is not a type-I
hypersensitivity reaction) and it does not respond to antihistamines. C4 levels fall
during an acute attack, while C3 levels may remain stable. It needs to be treated
with the inhibitor, present in fresh frozen plasma or C1 inhibitor concentrate;
concentrate is the preferred option. Recombinant preparations are not yet in routine
clinical use. 4

63-Immunoglobulin structure – which of the following regions forms the


antigen binding site?

The heavy chain


The light chain
The constant region of one heavy and one light chain
The hinge region
The variable region of one heavy and one light chain

The immunoglobulin is a Y shaped molecule that consists of four polypeptide


chains. Two chains are heavy and 2 are light. Each heavy and light chain has 2
regions, the constant and variable. The constant region is identical in all antibodies
of the same type. The antigen binding site is situated within the variable region of
the immunoglobulin structure, made up of both the heavy and light chains,
specifically within the hypervariable regions. 5

64-Which immunoglobulin can fix complement via the alternative pathway?

IgA
IgM
IgG
IgE
IgD

36
IgA is unusual in that it can fix complement via the alternative pathway. IgG and
IgM can fix complement via the classical pathway through the Fc portion of the
immunoglobulin. 1

65-Periodic fever can be associated with high levels of:

IgM
IgG
IgA
IgE
IgD

Hyper-IgD is associated with attacks of periodic fever every 4–8 weeks, with
each attack lasting 3–7 days. Other symptoms and/or signs include abdominal
pain, diarrhoea, vomiting, lymphadenopathy, arthralgia or arthritis and skin
lesions. IgD levels in periodic fever are raised and range from 145 to 5300 U/ml
(normal levels are < 100 U/ml). 5

66-Coeliac disease can be associated with low levels of:

IgM
IgG
IgA
IgE
IgD

There is an increased incidence of coeliac disease (also known as Gluten


Sensitivity Enteropathy (GSE)) in people with selective IgA deficiency. As
serological diagnosis of coeliac disease is based on the detection of IgA antibodies
to transglutaminase or of anti-endomysial antibody, it is important to check total
serum IgA before excluding the diagnosis on the basis of serology. 3

67-A 15-year-old boy presents with fever, rash and arthralgia. He has ++
protein and + blood on urinalysis. Investigations reveal the following:
erythrocyte sedimentation rate 32; C-reactive protein 12; full blood count
normal; and U+Es normal. Antistreptolysin-O titres are raised.

What immunological phenomenon is taking place?


37
Type-I hypersensitivity reaction
Type-II hypersensitivity reaction
Type-III hypersensitivity reaction
Type-IV hypersensitivity reaction
Cryoglobulinaemia

He has a poststreptococcal glomerulonephritis, an immune complex disease, which


is a type 3 hypersensitivity reaction. Other infectious causes of immune complex
disease include hepatitis B and C, cytomegalovirus and malaria. 3

68-A 13-year-old boy has suffered from recurrent sinus infections for the past
2 years and has a history of intermittent diarrhoea. He has a family history of
autoimmune disease – his mother has coeliac disease and sister autoimmune
haemolytic anaemia. His full blood count is normal.

What is his most likely diagnosis?

Coeliac disease
Systemic lupus erythematosus
Selective IgA deficiency
Common variable immunodeficiency
Wiskott–Aldrich syndrome

This boy most likely has common variable immune deficiency. The history of
invasive infection and gut involvement and the family history of immune
cytopaenia are suggestive. Selective IgA deficiency, although more common, is
often asymptomatic but it can also be associated with autoimmune disease. 4

69-An HIV positive child develops Pneumocystis jiroveci pneumonia. He is


deficient of what immunological component?

Complement
T cells
B cells
IgM

38
IgA

Patients with HIV have a deficiency of CD4 lymphocytes which are also known as
helper T cells. They are involved with antigen specific responses as well as
delayed type hypersensitivity. The risk of developing P. jiroveci pneumonia is
greatest with a CD4 count of 200 x 109/l or below. 2

70-A young boy has his second episode of Neisseria meningitis.

What is he deficient in?

C1
C2
C3
C4
C5

Opportunistic bacterial infections, such as meningococcus and gonorrhoea, occur


because of defects in the lytic complement pathway (C5–C9). The complement
system comprises a series of glycoproteins that are activated as part of a cascade
sequence. Activation of this cascade eventually leads to assembly of components
C5–C9 of the complement system. These form the membrane attack complex
(MAC), which forms a transmembrane channel that causes cell lysis by osmotic
shock. The pathway may be activated in a classic way, by antigen–antibody
immune complexes, apoptotic cells, C-reactive protein (CRP) bound to ligand, and
certain viruses and bacteria, or in an alternative way by bacterial endotoxin, fungal
cell walls, viruses, and certain tumour cells. Deficiencies of C3, C1q or factors H
or I lead to increased susceptibility to infection with capsulated bacteria. 5

71-Cells from a patient with severe combined immunodeficiency disease


(SCID) lack the adenosine deaminase enzyme.

This will have a direct effect on:

Synthesis of deoxynucleoside diphosphates (dNDPs)


Degradation of guanine nucleotides
Purine salvage

39
De novo synthesis of purine nucleotides
Degradation of adenine nucleotides

Lymphocytes need efficient salvage and interconversion pathways for purines and
pyrimidines during rapid bursts of proliferation, particularly in the lymphoid
germinal centres and fetal thymus. Adenosine deaminase (ADA) deficiency was
the first established cause of severe combined immunodeficiency disease (SCID),
the condition also being the first example of enzyme replacement therapy (initially
using red cell transfusions which contain ADA) in clinical medicine, and later
being the first disease to be treated by gene therapy, although with only partial
success. ADA has an important role in the intermediate pathways of purine
metabolism. Purine-nucleoside phosphorylase (PNP) is also active in this pathway,
but is a much rarer cause of SCID. It is interesting that deficiencies in both these
enzymes predominantly affect lymphocytes, despite their presence in most other
cells of the body. 3

72-Deranged cellular immunity:

Often presents with severe viral infections


Is associated with DiGeorge syndrome
Is associated with gram positive severe infections
Is associated with prematurity
Occurs in agammaglobulinaemia

Cell mediated immunity is mediated by T-cells and has, as one of it’s major
roles, the control of viral infections. Abnormalities of cell mediated immunity
occur in DiGeorge syndrome because of the absence of T-cell maturation
secondary to thymic abnormalities. Protection from bacterial infection is
classically associated with B-cell abnormalities and the consequent inability to
produce antibodies although such clear cut distinctions are slightly artificial.
Prematurity is associated with immaturity of the immune system but this mainly
manifests as abnormal antibody production. X-linked agammaglobulinaemia is a
pure B-cell defect and affected children have normal cell mediated immunity.
TTFFF

40
73-A boy was born at 26 weeks gestation. He had severe hyaline membrane
disease and was ventilated for 3 weeks. He is now three months old and has
just been discharged from hospital. He should receive pertussis vaccination
even if:

His Apgar scores were 3 at 1, and 5 at 5 minutes


He had a pneumothorax and 4 convulsions in the first week
He was hypoglycaemic and very jittery for a short period
After a pneumothorax, a cranial ultrasound scan showed an intraventricular
haemorrhage with some loss of cerebral substance
He continues to have uncontrolled convulsions two or three times a week

Children with cerebral damage, a personal history of convulsions, or a family


history of febrile convulsions are at increased risk of a febrile fit following
pertussis and measles immunisation. However, these are not absolute
contraindications and they are not at any greater risk of permanent adverse effects
from the vaccines and should receive them. Children whose epilepsy is poorly
controlled should not be immunised until control is achieved, although convulsions
themselves are not a contraindication.

The only absolute contraindications for pertussis-containing vaccines are:

 Confirmed anaphylactic reaction to a previous dose of pertussis-containing


vaccine, or
 Confirmed anaphylactic reaction to neomycin, streptomycin or polymyxin B
(which may be present in trace amounts in the vaccine). T T T T F

74-The following are live vaccines ...

BCG
Measles
Oral polio
Pertussis
Rubella

41
IM (Salk) polio vaccine is inactivated, while oral polio is an activated live vaccine.
The pertussis vaccine is acellular and made from part of the pertussis cell.
TTTFT

75-With regard to the Measles, Mumps and Rubella (MMR) vaccine:

It should not be given if there is a history of previous measles infection


It should not be given if there has been a previous measles immunisation
It is contraindicated in a child on oral corticosteroids
It is contraindicated where there is a personal history of febrile convulsions
It is contraindicated if there is a family history of epilepsy

Live vaccine is contraindicated in children who may be immunosuppressed (such


as oral steroids). Steroids, such as prednisolone, given to children either orally or
rectally at a daily dose of 2mg/kg/day for at least one week or 1mg/kg/day for one
month should be deemed as a contraindication and administration of live vaccines
should be postponed for at least 3 months after immunosuppressive treatment has
stopped.

Previous infection or immunisation is not a contraindication, and 2 MMR doses are


recommended. A fit within 72 hours of a previous dose would be a
contraindication, although febrile convulsions and a family history of epilepsy are
not. FFTFF

76-Which of the following pathogens are associated with the accompanying


immune disorder?

Mycobacterium and cellular immunity


Gram -ve bacteria and complement deficiency
Enterovirus and antibody defects
Staphylococcus and complement deficiency
Haemophilus influenzae and neutrophil defect

The pathogens associated with an underlying immune disorder are:

 Mycobacterium – cellular immunity


 Gram -ve and Gram +ve bacteria – neutrophil defects

42
 Enterovirus – antibody defects
 Staphylococcus – complement deficiency
 Neisseria – complement deficiency
 Haemophilus influenzae – antibody defects
 Salmonella – type 1 cytokine defects and cell-mediated defects
 Mycoplasma – antibody defects
 Herpes virus – defects in cell-mediated immunity. TFTTF

77-Which of the following statements regarding chronic cutaneous candidiasis


are true?

It is associated with poor T-cell function


It can be successfully treated with bone marrow transplantation
It is totally eradicated with fluconazole
It can cause failure to gain weight in infancy
It may be associated with an endocrinopathy in adult life

The exact underlying defect in chronic cutaneous candidiasis is still poorly


understood; there is, however, poor T-cell proliferation and cytokine production.
There is persistent Candida infection, which is recurrent and not eliminated by
antifungal treatment. Oesophageal involvement can lead to gastro-oesophageal
reflux and dropping off the weight centiles. In over 50% of cases, there is, in the
second to third decade of life, an endocrinopathy that may involve combinations of
hypoparathyroidism, Addison’s disease, pernicious anaemia, hypothyroidism
and diabetes mellitus. T T F T T

78-Which of the following children should not receive live vaccines?

Those requiring regular intravenous immunoglobulin therapy


Those on a prednisolone dose of 3 mg/kg per day for longer than 1 week
Children with graft-versus-host disease
Children on chemotherapy
Children who have had a solid organ transplant

Intravenous immunoglobulin treatment will negate the live virus. The other
categories bestow vulnerability and could lead to evolution of the disease. Post-
transplant children are maintained on lifelong immunosuppression. 4 T

43
79-In which of the following diseases should a complement abnormality be
suspected?

Recurrent folliculitis
Pneumocystis carinii infection
Recurrent swelling after trauma
Delayed separation of the cord
Pancreatitis

A decreased C3 level is seen in:

 Systemic lupus erythematosus (antinuclear antibodies (ANA), antidouble-


stranded DNA)
 Mesangiocapillary glomerulonephritis
 Membranoproliferative glomerulonephritis (type 2)
 Post-streptococcal glomerulonephritis
 Infective endocarditis
 Infected ventricular shunts
 Chronic Gram-negative infection.

Increased complement activation is seen in:

 Hereditary angioneurotic oedema (C1 inhibitor deficiency)


 Pregnancy
 Paroxysmal nocturnal haemoglobinuria
 Glomerulonephritis grade 2.

The level is not changed in focal sclerosis, focal segmental glomerulonephritis,


Henoch–Schönlein purpura or haemolytic-uraemic syndrome.

A defect in the complement system should be suspected in:

 Focal swelling after trauma


 Recurrent meningococcal infection (C5–C9) (encapsulated organisms)
 Recurrent staphylococcal infection
 Lupus-like syndrome (major histocompatibility complex, chromosome 6).

Complement abnormality is not suspected in recurrent folliculitis, Pneumocystis


carinii infection (combined immunodeficiency) or delayed separation of the cord

44
(leukocyte adhesion defect), or inherent complement deficiency in
membranoproliferative disorders. Pancreatitis involves the consumption of normal
levels of complement. F F T F F

80-This substance is derived from arachidonic acid. The arachidonic acid is


cleaved into cyclooxygenase derivatives and lipoxygenase derivatives. This
substance is produced by the latter and is synthesised by leukocytes and mast
cells. Leukocytes and mast cells are mediators of inflammation and allergic
reactions by increasing chemotaxis and vascular permeability and inducing
smooth muscle constriction. They also cause arteriolar constriction and
bronchoconstriction.

What is the substance?

Thromboxane
Prostacyclin
Prostaglandin
Leukotriene
Eosinophil

Leukotrienes are synthesised by leukocytes and mast cells. They mediate


inflammation and allergic reactions in the following ways:

 By increasing neutrophil/eosinophil chemotaxis and vascular permeability,


and inducing smooth muscle constriction
 By causing arteriolar constriction and bronchoconstriction.

Leukotriene D4 is identified as the slow-reacting substance of anaphylaxis (SRS-


A) and is responsible for smooth muscle contraction. Leukotrienes are derived
from arachidonic acid and not palmitic acid. The arachidonic acid is cleaved into
cyclooxygenase derivatives, which produce prostaglandins, prostacyclins and
thromboxanes. The other pathway produces leukotrienes. Leukotrienes do not act
synergistically with SRS-A as leukotrienes are produced by mast cells to dampen
the inflammatory response and decrease granulocyte formation at the site of
invasion. 4

45
81-An 18-month-old male presents with recurrent pneumonia, ear infections
and tonsillitis. The family history includes a previous male infant death at 2
years of age from pneumococcal meningitis. The parents are therefore very
concerned and wish further investigations to be carried out.

After the results have returned, it is found that the child has absent or
decreased levels of immunoglobulins A, G and M. He has reduced responses
to blood group antibodies to immunisations and an increased percentage of E
rosettes with red blood cells. His PHA and nitroblue tetrazolium (NBT) tests
are normal.

The following differential diagnosis may be considered, but the ultimate


disorder is which of the following?

Common variable immune deficiency


X-linked agammaglobulinaemia
Chronic granulomatous disease
Severe combined immunodeficiency syndrome
Chédiak–Higashi syndrome

X-linked agammaglobulinaemia is the most common immunoglobulin deficiency.


The gene is localised to the long arm of the X chromosome, with the abnormality
being in the gene for the B-cell tyrosine kinase. It is more common in males. This
disorder is associated with an increased risk of blood malignancies. It presents after
3–6 months of age when the level of IgG resulting from in utero placental
transfer has decreased. The child presents with recurrent bacterial infections in the
first 2 years of life, namely lung and sinus infections. Prenatal diagnosis is
possible. Bone marrow transplantation is carried out if medical interventions fail. 2

82-A 4-year-old, fair-haired, fair-skinned child presents to the outpatient


department with a history of recurrent chest infections, ataxia and muscle
weakness. He has a long history of fatigue and malaise, and a GP has
wondered whether he suffers from infectious mononucleosis. He is currently
on sodium valproate for seizures, which have been observed since the age of 2
years. On examination, he has bilateral nystagmus, photophobia and an
increased red light reflex. He shows signs of sensory and motor neuropathy,
and marked muscle weakness.

46
Which of the following is the most likely informative investigation to be
undertaken?

Full blood count


Clotting profile
Bacterial and viral serology
Chromosomal studies
White cell count and natural killer cell function
tests

Investigations depict a decreased antibody-dependent cell-mediated cytolysis of


the tumour-like reaction. There is decreased function of natural killer cells, a
prolonged bleeding time, impaired platelet aggregation, decreased neutrophil
number and a pancytopenia when associated with bacterial and viral illnesses.

The suspected diagnosis here is Chediak Higashi syndrome. Affected children


present with ataxia, seizures, muscle weakness and an illness similar in picture to
lymphoma with lymphotropic viruses, e.g. Epstein Barr virus. Ocular symptoms
include nystagmus photophobia and an increased red reflex. 5

83-A 4-year-old, fair-haired, fair-skinned child presents to the outpatient


department with a history of recurrent chest infections, ataxia and muscle
weakness. He has a long history of fatigue and malaise, and a GP has
wondered whether he suffers from infectious mononucleosis. He is currently
on sodium valproate for seizures, which have been observed since the age of 2
years. On examination, he has bilateral nystagmus, photophobia and an
increased red light reflex. He shows signs of sensory and motor neuropathy,
and marked muscle weakness.

What is the most probable diagnosis?

Wilms’ tumour
Chédiak–Higashi syndrome
Infectious mononucleosis
Epstein–Barr virus-driven Burkitt’s lymphoma
Guillain–Barré syndrome

47
This autosomal recessive disorder occurs due to dysfunctional and reduced natural
killer cells. Pathologically, the leukocytes contain giant cytoplasmic granules.
Other cells can also contain these granules, giving features of the disease.
Melanocytes do not, however, disperse pigments within the cells; hence there are
features of partial albinism. There are no hyperpigmented lesions of the
lumbosacral area. Schwann cells are affected, causing sensory and motor
neuropathies. People with this disorder have an increased susceptibility to
infections due to defective chemotaxis, degranulation and bactericidal activity of
the neutrophils. There is an increased incidence of malignancies and a high
mortality rate with respect to infection.

Affected children present with ataxia, seizures, muscle weakness and an illness
similar in picture to lymphoma with lymphotrophic viruses, e.g. Epstein–Barr
virus. Ocular symptoms include nystagmus, photophobia and an increased red
reflex. 2

84-A 4-month-old male infant is referred with a chronic history of superficial


skin abscesses and painful lymphadenopathy. Routine investigations reveal
chronic anaemia and neutrophilia with a raised erythrocyte sedimentation
rate, and Grampositive cocci have grown in a pus sample obtained from a
perianal abscess 4 weeks previously. On examination, you find a pale child
with a marked swelling of the left submandibular region. He has reduced air
entry with crepitations at the left lower lobe consistent with a left lower lobe
pneumonia. Abdominal examination reveals hepatomegaly and previous
scarring from peri-anal abscesses. Routine sputum and stool samples are sent
from the clinic, which reveal that he has grown Serratia marcescens from his
sputum sample. He has had Salmonella infection in the previous 3 months.

Which is the most informative investigation to consider?

Immunoglobulin levels
Skin biopsy of an abscess lesion
Nitroblue tetrazolium (NBT) test
Clotting screen
Neutrophil function test

Chronic granulomatous disease may manifest soon after birth with superficial skin
sepsis and painful regional lymphadenopathy. Lesions heal incompletely with
sinus formation. Due to the inability to kill catalasepositive micro-organisms,
48
chronic deep infections occur, and common recurrent skin abscesses with
Staphylococcus, Escherichia coli, Serratia, Klebsiella and Candida may be seen.
Less common infections include those involving Shigella, Salmonella and
Pseudomonas.

In the NBT test the colourless nitroblue tetrazolium is reduced to blue formazan by
the activity of the phagocyte oxidase (phox) enzyme system. Flow cytometry can
provide quantification of the activity in the cells. A negative NBT test (remains
colourless) may confirm the diagnosis of chronic granulomatous disease. 3

85-A 4-month-old male infant is referred with a chronic history of superficial


skin abscesses and painful lymphadenopathy. Routine investigations reveal
chronic anaemia and neutrophilia with a raised erythrocyte sedimentation
rate, and Grampositive cocci have grown in a pus sample obtained from a
perianal abscess 4 weeks previously. On examination, you find a pale child
with a marked swelling of the left submandibular region. He has reduced air
entry with crepitations at the left lower lobe consistent with a left lower lobe
pneumonia. Abdominal examination reveals hepatomegaly and previous
scarring from peri-anal abscesses. Routine sputum and stool samples are sent
from the clinic, which reveal that he has grown Serratia marcescens from his
sputum sample. He has had Salmonella infection in the previous 3 months.
You have instigated a Nitroblue tetrazolium (NBT) test.

The diagnosis is which of the following?

Chédiak–Higashi syndrome
Chronic granulomatous disease
Lactoferrin deficiency
Shwachman syndrome
Systemic lupus erythematosus

Chronic granulomatous disease may manifest soon after birth with superficial skin
sepsis and painful regional lymphadenopathy. Lesions heal incompletely with
sinus formation. Due to the inability to kill catalasepositive micro-organisms,
chronic deep infections occur, and common recurrent skin abscesses with
Staphylococcus, Escherichia coli, Serratia, Klebsiella and Candida may be seen.
Less common infections include those involving Shigella, Salmonella and
Pseudomonas.

49
All these pathogens display a lack of oxidative respiratory burst and produce
oxygen free radicals. The neutrophil is unable to activate one or more oxidases to
consume oxygen to produce superoxide or hydrogen peroxide following
phagocytosis. Superoxide stimulates the hexose monophosphate shunt, and halides
are produced. The interaction of reactive oxygen molecules, myeloperoxidase and
halides with phagocytic vacuoles results in the effective killing of catalase-
containing bacteria. Gram-negative rods and catalase-negative organisms are killed
normally as they generate hydrogen peroxide. Aspergillus is a common secondary
pathogen, but herpesvirus is not found as a pathogen.

Other clinical manifestations include respiratory tract infections, sinusitis, urinary


tract infections, obstructive uropathy and liver infections (abscesses) with
hepatosplenomegaly. Gastrointestinal involvement includes necrotic granulomas,
colitis, oesophageal strictures, perinatal abscess and pyloric strictures. Chronic
osteomyelitis of the small bones of the hands and feet may be difficult to treat.

The differential diagnosis includes neutrophil-killing defects,


Chédiak–Higashi syndrome, lactoferrin deficiency, mobility disorders such as
lazy leukocyte syndrome and Shwachman syndrome. Chronic illnesses, including
systemic lupus erythematosus, rheumatoid arthritis, tuberculosis, toxoplasmosis,
hyper-IgE syndrome and malnutrition, may also be associated. 2

86-A 4-month-old male infant is referred with a chronic history of superficial


skin abscesses and painful lymphadenopathy. Routine investigations reveal
chronic anaemia and neutrophilia with a raised erythrocyte sedimentation
rate, and Grampositive cocci have grown in a pus sample obtained from a
perianal abscess 4 weeks previously. On examination, you find a pale child
with a marked swelling of the left submandibular region. He has reduced air
entry with crepitations at the left lower lobe consistent with a left lower lobe
pneumonia. Abdominal examination reveals hepatomegaly and previous
scarring from peri-anal abscesses. Routine sputum and stool samples are sent
from the clinic, which reveal that he has grown Serratia marcescens from his
sputum sample. He has had Salmonella infection in the previous 3 months.

After a Nitroblue tetrazolium (NBT) test, the diagnosis has been confirmed as
chronic granulomatous disease.

Appropriate management on discharge may be which of the following?

50
Prophylactic antibiotics
Amphotericin
White cell and granulocyte infusions
Interferon-γ infusions
All of the above

Chronic granulomatous disease may manifest soon after birth with superficial skin
sepsis and painful regional lymphadenopathy. Lesions heal incompletely with
sinus formation. Due to the inability to kill catalasepositive micro-organisms,
chronic deep infections occur, and common recurrent skin abscesses with
Staphylococcus, Escherichia coli, Serratia, Klebsiella and Candida may be seen.
Less common infections include those involving Shigella, Salmonella and
Pseudomonas.

Antimicrobial prophylaxis along with early and aggressive treatment of infections


are the basis of treatment, with interferon used in acute infection (commonly used
daily in the US but in Europe is not used routinely due to expense). 5

87-A 4-month-old male infant is referred with a chronic history of superficial


skin abscesses and painful lymphadenopathy. Routine investigations reveal
chronic anaemia and neutrophilia with a raised erythrocyte sedimentation
rate, and Grampositive cocci have grown in a pus sample obtained from a
perianal abscess 4 weeks previously. On examination, you find a pale child
with a marked swelling of the left submandibular region. He has reduced air
entry with crepitations at the left lower lobe consistent with a left lower lobe
pneumonia. Abdominal examination reveals hepatomegaly and previous
scarring from peri-anal abscesses. Routine sputum and stool samples are sent
from the clinic, which reveal that he has grown Serratia marcescens from his
sputum sample. He has had Salmonella infection in the previous 3 months.
After investigation, the diagnosis has been confirmed as chronic
granulomatous disease.What is the pattern of inheritance?

Autosomal dominance inheritance


Autosomal recessive inheritance
X-linked recessive inheritance
X-linked dominant inheritance
Co-dominant inheritance

51
Chronic granulomatous disease may manifest soon after birth with superficial skin
sepsis and painful regional lymphadenopathy. Lesions heal incompletely with
sinus formation. Due to the inability to kill catalasepositive micro-organisms,
chronic deep infections occur, and common recurrent skin abscesses with
Staphylococcus, Escherichia coli, Serratia, Klebsiella and Candida may be seen.
Less common infections include those involving Shigella, Salmonella and
Pseudomonas. All these pathogens display a lack of oxidative respiratory burst and
produce oxygen free radicals. The neutrophil is unable to activate one or more
oxidases to consume oxygen to produce superoxide or hydrogen peroxide
following phagocytosis. Superoxide stimulates the hexose monophosphate shunt,
and halides are produced. The interaction of reactive oxygen molecules,
myeloperoxidase and halides with phagocytic vacuoles results in the effective
killing of catalase-containing bacteria. Gram-negative rods and catalase-negative
organisms are killed normally as they generate hydrogen peroxide. Aspergillus is a
common secondary pathogen, but herpesvirus is not found as a pathogen. Other
clinical manifestations include respiratory tract infections, sinusitis, urinary tract
infections, obstructive uropathy and liver infections (abscesses) with
hepatosplenomegaly. Gastrointestinal involvement includes necrotic granulomas,
colitis, oesophageal strictures, perinatal abscess and pyloric strictures. Chronic
osteomyelitis of the small bones of the hands and feet may be difficult to treat. The
differential diagnosis includes neutrophil-killing defects, Chédiak–Higashi
syndrome, lactoferrin deficiency, mobility disorders such as lazy leukocyte
syndrome and Shwachman syndrome. Chronic illnesses, including systemic lupus
erythematosus, rheumatoid arthritis, tuberculosis, toxoplasmosis, hyper-IgE
syndrome and malnutrition, may also be associated. F T T F F

88-A 15-year-old girl presents to the outpatient clinic with a chronic history of
abdominal pain at the time of her menstrual cycle. Her parents have
wondered whether she has been in contact with any environmental pollens or
grasses, as at times she has a hoarse voice and her skin comes out in mild
oedematous swellings. The parents feel that this could be a psychosomatic
element, as it often occurs at times of illness, stress and cold. On examination,
there is no fever, rash, arthropathy or oedema. Cardiovascular, respiratory
and abdominal examination is unremarkable.

A possible diagnosis could be which of the following?

Idiopathic chronic urticaria


Systemic lupus erythematosus

52
Autoimmune Henoch–Schönlein purpura
Kawasaki’s disease
Hereditary angioneurotic oedema

Hereditary angioneurotic oedema is caused by a deficiency or a defect in C1


esterase inhibitor protein, the amount found being <25% the normal value. The
protein modulates the intravascular activation of complement, and its deficiency
leads to angioedema. There is an inability to synthesise normally functioning C1
inhibitor. The condition is genetically inherited in an autosomal dominant fashion.
This disorder usually presents in late childhood or adolescence, ie aged 10–20,
when attacks become more frequent. Hereditary angioneurotic oedema causes 2%
of all angioedemas. It is found that 85% of individuals have very low levels of
inhibitor and 15% have normal or raised non-functioning protein; this may
therefore be used in a functional assay. This disorder involves the deeper layers of
the skin. Clinically, hereditary angioneurotic oedema presents with:

 Abdominal pain and cramps due to swelling of the intestinal tissues


secondary to visceral oedema
 Subcutaneous oedema and swelling (72 hours maximum)
 Laryngeal oedema (presenting as a hoarse voice and sometimes causing fatal
obstruction); these may be unresponsive to antihistamines
 Symptoms related to phases of the menstrual cycle
 Triggers including a prodromal rash, skin trauma (as the protein involved is
within the fibrin cascade), menstruation and emotional stress. There is no
urticarial rash or inflammation suggestive of pruritus.

Investigations

During an attack, C4 and C2 levels are decreased. There is an excessive


stimulation of C2, C4 and related vasopeptides, due to uninhibited activity of
complement. The deficiency of C1 inhibitor permits C1 to continuously cleave its
substrates C4 and C2 to cause them to become deficient. The C1 and C3 levels are
normal, as is the number of B cells. C2b splits off to become the vasoactive
component.

Treatment

Treatment used to date includes anabolic steroids, to increase protein levels. This
prevents acute attacks, and the symptoms resolve in 2–3 days. Fresh frozen
plasma and C1 esterase concentrate are used in the acute phase to restore normal
53
levels of inhibitor. Androgen agonists, ie danazol or stanozolol, are used in the
chronic phase. The side-effects of danazol include androgenic problems in
childhood. Other drugs used include alpha-aminocaproic acid and tranexamic acid.
Steroids, adrenaline and antihistamines are not effective. 5

89-Theme: Complement receptor abnormalities

A CR3 complement receptor


B Alternative pathway (C5, C6, C7, C8, C9)
C C1 esterase inhibitor
D Decrease in complement receptor (CR2)
E Alternative pathway (C3 convertases)
F Alternative pathway (delay accelerating factor (DAF))
G Classical pathway (C234)
H Classical pathway, mannose-binding lectin
I C1Q, C1R, C2, C3, C4
J Properdin

Match the complement abnormalities above to the following diseases:

Scenario 1 Pyogenic bacterial infection

CR3 complement receptor Correct answer

Scenario 2 Glomerulonephritis-associated systemic lupus erythematosus (SLE)

C1Q, C1R, C2, C3, C4 Correct answer

Scenario 3 Recurrent neisserial infection

Alternative pathway (C5, C6, C7, C8, C9) Correct answer

Scenario 4 Non-recurrent neisserial infection

Properdin Correct answer

Scenario 5 Susceptibility to infection

54
Classical pathway, mannose-binding lectin Correct answer

Scenario 6 Hereditary angioneurotic oedema

C1 esterase inhibitor Correct answer

Scenario 7 Recurrent pyogenic infections

CR3 complement receptor Correct answer

Scenario 8 Renal diseases

Alternative pathway (C3 convertases) Correct answer

Scenario 9 Paroxysmal nocturnal haemoglobinuria

Alternative pathway (delay accelerating factor (DAF)) Correct answer

Scenario 10 Pyogenic infections

Decrease in complement receptor (CR2) Correct answer

The following table summarises the complement proteins and the diseases with
which they are associated.

Pathway Complement protein Disease(s)


Classical C1q, C1R, C2, C4 SLE, glomerulonephritis
Recurrent neisserial
Alternative D
infections
Non-recurrent
Properdin
meningococcal infections
Susceptibility to infections
Lectin Mannose-binding lectin (MBL)
(otitis media)
C3 deficiency SLE, severe infections,
C3, factor I, factor H
syndrome glomerulonephritis

55
Membrane Recurrent neisserial
C5, C6, C7, C8, C9
attack infections
C1 inhibitor, delay accelerating
Complex control Hereditary angioneurotic
factor (DAF), homologous restriction
proteins oedema
factor (HRF)
Complement Recurrent pyogenic
CR3
receptor infections

C1 activity is increased in C1 inhibitor deficiency. C3a, C4a and C5a all have the
ability to bind to mast cells and leukocytes to trigger the release of histamine and
mediators. They are involved in anaphylaxis, vasodilatation, swelling and
inflammation, bee-sting anaphylaxis and extrinsic allergic alveolitis. C5a is a
potent neutrophil chemotactic agent that promotes macrophages to CR1 and CR3
receptors to neutralise. C3a mediates the suppression of antibody responses. C3b
enhances cellmediated toxicity and solubilisation of the immune complexes. Low
C3 and CH50 levels correlate strongly with a very poor outcome in patients with
multiorgan failure. C5b enhances the antibody response involved in the chemotaxis
of neutrophils, monocytes and eosinophils. Individuals with a deficiency of C5 are
at an increased risk of developing meningococcal sepsis.

The membrane attack complex (C5–C9)

The membrane attack complex (MAC) assembles following the cleavage of C5 to


form C5b via the classical or alternative pathway. It inserts into cell membranes to
form a channel that traverses the membrane and leads to cell lysis. Its binding to
cells is inhibited by very low-density lipoprotein, and autologous cells contain
membrane control proteins that partially protect them from the action of MAC.
MAC is active only against autologous cells. Paroxysmal nocturnal
haemoglobinuria is caused by a decreased number of control membrane proteins so
there is an increased sensitivity to MAC.

Complement receptors

These receptors participate in various functions of the host defence system,


including the C1 and C4 neutralisation of viruses. There are four complement
receptors:

CR1: found in leukocytes in tissues and the circulation. It has a role in immune
complex handling and is decreased in SLE as an acquired defect.

56
CR2: important in the lymphocyte reaction with other cells. Epstein–Barr virus
may act as a ligand.

CR3 and CR4: form a receptor that also includes IFA-1 (a lymphocyte receptor).
If the level of this receptor is decreased, pyogenic infection results as the
neutrophils are unable to bind the bacteria coated in C3. Cryoglobulinaemia
displays a low C4 but a normal C3 level.

90-Theme: Human leukocyte antigens

A HLA DR2/W2, DR/W2, B8


B HLA DR3
C HLA DQW2, DR3/7
D HLA A28
E HLA B5
F HLA B27
G HLA DR5
H HLA A3/B14
I HLA B8/DR3
J HLA DR4

Scenario 1 Which of the above HLA associations is responsible for ankylosing


spondylitis, psoriatic arthropathy, Reiter’s disease and reactive arthritis? This
reactive arthritis may occur after a gut infection with Salmonella, Shigella,
Yersinia enterocolitica or Campylobacter and thus presents with bloody diarrhoea.

HLA B27 Correct answer

Scenario 2 Which of the above HLA associations is responsible for Behçet’s


disease and the arthritis of inflammatory bowel disease? Ten per cent of suffers of
inflammatory bowel disease present with arthritis, and some therefore have
associated ankylosing spondylitis.

HLA B5 Correct answer

Scenario 3 Which of the above HLA associations would be suggestive in a 3-year


old child who has been weaned on to a gluten-containing diet and subsequently
suffers from abdominal protrusion, buttock-wasting and steatorrhoea, and presents
with positive anti-gliadin antibodies.

57
HLA DQW2, DR3/7 Correct answer

HLA associations

HLA DR/W2,
Systemic lupus erythematosus
DR/W3 and B8
HLA B8/DR3 Dermatomyositis herpetiformis
HLA DR5/DR2 Kaposi’s sarcoma
Chronic active hepatitis, Graves’ disease, myasthenia gravis,
HLA DR3 Addison’s disease and Sjögren’s syndrome, idiopathic
membranous nephropathy, systemic lupus erythematosus
Goodpasture syndrome, narcolepsy, multiple sclerosis, juvenile
HLA DR2
insulindependent diabetes mellitus
HLA DQW2,
Coeliac disease
DR3/7
HLA A3/B14 Idiopathic haemochromatosis
HLA A28 Schizophrenia
Behçet’s disease and arthritis of inflammatory bowel disease
HLA B5
(IBD) (ulcerative colitis)*
Ankylosing spondylitis, psoriatic arthropathy**, Reiter’s
HLA B27
disease, reactive arthritis***
HLA DR4 Rheumatoid arthritis
HLA DR5 Hashimoto’s disease, pernicious anaemia
HLA B35 Subacute thyroiditis
HLA DR2, 3, 4 Insulin-dependent diabetes mellitus
HLA A1, B8, Linkage equilibrium in nephrotic syndrome, acquired
DR3 immunodeficiency syndrome (AIDS)

*IBD arthritis:

 10% of IBD sufferers get arthritis. Some get ankylosing spondylitis


associated with HLA B27

**Psoriasis May:

 show a weak HLA B27 link

58
***Reactive arthritis:

 After Salmonella, Shigella, Yersinia enterocolitica, Camplyobacter and


Reiter’s disease, i.e. causes of bloody diarrhoea

91-Theme: Hypersensitivity

Match the following types of hypersensitivity with the three clinical scenarios
given:

A Type I – immediate and anaphylactic hypersensitivity


B Type II – cell-bound hypersensitivity
C Type III – immune complex hypersensitivity
D Type IV – delayed T-cell hypersensitivity
E Type V – stimulatory hypersensitivity
F Type VI – killer hypersensitivity
G Type VII
H Type VIII
I Type IX
J Type X

Scenario 1 A 3-year-old boy is currently undergoing stem cell bone marrow


transplantation due to severe combined immunodeficiency. On examination, he is
found to have an erythematous maculopapular blanching rash with marked
erythema of the palms of the hands and soles of the feet. He is found to have
associated abdominal pain, diarrhoea and jaundice. The boy is currently in his third
week of treatment, and his recent phytohaemagglutinin (PHA) test confirms that
his bone marrow has grafted. You are suspicious that he may be suffering from
veno-occlusive disease with an associated rejection abnormality.

Which type of hypersensitivity is involved?

Type IV – delayed T-cell hypersensitivity Correct answer

Scenario 2 A 12-year-old girl presents to the emergency department with a


nonblanching purpuric rash over her lower limbs and buttocks. She is complaining
of a mild abdominal pain and a virus-like illness. On examination, she is found to
have red-purple purpura throughout her lower abdomen and posteriorly over her
buttocks. She is also found to have swelling of the left ankle and right knee. Her
investigations show her to have ++ protein and +++ blood in her urine; growth has,

59
however, proven negative. The girl’s blood pressure is currently normal, as are
her other observations. You wonder whether this child is suffering from
meningococcaemia or a vasculitis.

What type of hypersensitivity is involved?

Type III – immune complex hypersensitivity Correct answer

Scenario 3 A child presents to the outpatient department with exophthalmia,


biorbital proptosis, lid lag and loss of weight. Her thyroid function tests prove her
to have a raised thyroxine (T4) level, and she is suspected of having
hyperthyroidism.

Which type of hypersensitivity reaction is involved?

Type V – stimulatory hypersensitivity Correct answer

There are six types of hypersensitivity:

Type 1 – This is an immediate and often anaphylactic reaction involving IgE,
IgG4, mast cells and basophils. It involves a vasodilatory response, activation of
eosinophils and leukotrienes, and the release of vasodilators such as histamine,
serotonin and bradykinins. Histamine is a spasmogen, causing bronchial smooth
muscle contraction. A wheal and flare is formed. The process is involved in:

 Atopic disease
 Asthma
 Hay fever (pollen)
 Eczema
 Anaphylaxis
 Food and drug intolerance
 Bee and wasp stings.

For the purpose of exams, remember that sodium cromoglicate stabilises the mast
cell membrane.

Type 2 – This is cell-bound hypersensitivity, which is antibody dependent and


involves the cytotoxic cells. Here, circulating antibody (IgG and IgM) binds to
cell–cell surface antigen receptors and activates complement, which causes cell

60
lysis/damage and macrophage opsonisation with neutrophil activation. This type of
hypersensitivity is involved in:

 Graves’ disease
 Haemolytic anaemia
 Myasthenia gravis
 Idiopathic thrombocytopenic purpura (ITP)
 Goodpasture syndrome
 Hyperacute graft rejection
 Reduction of ABO incompatibility
 Role in cancer
 Blockage of antibody.

Type 3 – This involves circulating immune complexes and is complement


mediated. It is activated via IgG and IgA. There are three mechanisms for
producing disease:

1. The combined effects of low fever and a weak antibody response


2. As a complication of autoimmune disease in which the continued production
of antibody to self antigen forms complexes
3. The formation of immune complexes at the body surfaces, e.g. in the lungs.

Type 4 – This involves the cell-mediated immune system. It involves a delayed
T-cell-mediated reaction. It is associated with:

 Acute early rejection


 Graft-versus-host disease (late rejection)
 Rheumatoid arthritis
 Contact dermatitis
 Acute allergic alveolitis
 The reaction in tuberculosis with the tuberculin skin test (72 hours) and
granulomatosis (21–28 days)
 The Heaf test response.

This delayed hypersensitivity is seen with:

 Sarcoidosis
 Malignant lymphomas such as Hodgkin’s disease
 DiGeorge syndrome (absent or decreased T-cells)
 Corticosteroid treatment
 Leprosy (borderline)
61
 Malnutrition
 Wiskott–Aldrich syndrome
 Crohn’s disease
 Extrinsic allergic alveolitis
 Schistosomiasis
 Extreme old age – it decreases with age
 It is not associated with the Arthus reaction.

Type 5 – This is stimulatory with IgG antibody, which results in:

 Long-acting thyroid stimulation, which causes the prolonged stimulation of


thyroid hormone
 Graves’ diseases and hyperthyroidism.

Type 6 – This class involves killer cells, which lyse targets coated with
antibody, e.g. tumours or helminths.

Note: Types 7–10 do not exist

92-Regarding tuberculin testing:

It traditionally involves an injection into the extensor surface of the left forearm
The Heaf test should ideally be read between 48 and 72 hours
A positive result occurs when the area of induration is > 5 mm
Is negative if the Heaf grade is ‘1’
Induration > 15 mm requires further investigation and possible antituberculous
chemotherapy

Tuberculin testing traditionally involves an injection into the flexor surface of the
left forearm. The Heaf test is ideally read at 7 days (between 3 and 10 days) and
the Mantoux test is read at 48–72 hours (but up to 96 hours). A positive result
occurs when the area of induration is > 5 mm. N.B. the area of ‘flare’ is
irrelevant.

The Heaf test is graded 0–4. Heaf grades 0–1 are negative and grades 2–4
are positive. Strongly positive reactions (i.e. Heaf grade 3–4 or induration > 15
mm) require further investigation and possible antituberculous chemotherapy.
FFTTT

62
93-Theme: Immunisation

A No live vaccines

B No vaccines at all

C Normal immunisation schedule D

D Normal schedule with separate pertussis vaccine

E Normal schedule without BCG (Bacilel Camille–Guérin)

F Normal schedule and unconjugated pneumococcal vaccination

G Normal schedule without polio vaccine

H Normal schedule with substitution of single vaccines for MMR (measles,


mumps, rubella)

I Normal schedule without MMR

J Normal schedule without MMR or BCG

For each of the following children, choose the most appropriate immunisation
policy from the above list. Each item may be used once, more than once or not at
all.

Scenario 1 A child born in the UK with vertically acquired human


immunodeficiency virus (HIV) infection.

Normal schedule without MMR or BCG Correct answer

Some experts advise that measles vaccine can be given if the child has a CD4
count > 200. Mumps and rubella vaccines can be given singly.

Scenario 2 A child diagnosed on the autistic spectrum.

Normal immunisation schedule D Correct answer

Autism is not a contraindication to any immunisation.

63
Scenario 3 A child who has sickle cell trait.

Normal immunisation schedule D Correct answer

Children with sickle cell trait should receive the universal schedule. Those with
sickle cell disease should receive the unconjugated vaccine at age 2-3 years, in
addition to the conjugated vaccine as part of the universal schedule (see
http://www.immunisation.nhs.uk/Immunisation_Schedule).

Note that BCG is NOT part of the UK’s universal schedule but may be offered
in areas of high TB incidence.

94-With regard to Bacillus Calmette-Guérin (BCG):

High-risk infants should have a positive Heaf test before immunisation proceeds
It is safe in asymptomatic HIV-positive patients
It may be given at the same time as other live vaccines
It should be given after a positive tuberculin test
Offers some protection against leprosy

High-risk infants do not require skin testing before immunisation up to the age of 3
months. BCG should be given only after a negative tuberculin test. It can be given
at the same time as other live vaccines; otherwise a gap of 3 weeks should be
observed. There is a risk that a suboptimal response to both may occur if this gap is
not observed. Live oral polio vaccine (OPV), which works by inducing gut
immunity, is the exception and can be given at any time. HIV infection is an
absolute contraindication to BCG vaccination. F F T F T

95-The following are notifiable diseases:

Acquired immune deficiency syndrome (AIDS)


Mumps
Tuberculosis (TB)
Rubella
Malaria

Mumps, TB, rubella and malaria are all notifiable diseases. F T T T T


64
96-IgA deficiency:

is the commonest form of primary immunodeficiency


usually presents as growth faltering
has an association with coeliac disease
is a contraindication to BCG (bacille Calmette–Guérin) immunisation
increases the risk of transfusion reactions

IgA deficiency is relatively common, affecting between 1 in 300 and 1 in 600


people. Most cases are subclinical in childhood, though they may present with
atopy or recurrent infections of the respiratory tract or gastrointestinal system.
There are known associations with autoimmune disorders, coeliac disease,
Crohn’s disease, ulcerative colitis and malignancies in older patients. Many
patients will develop antibodies against immunoglobulin A (IgA), and are at
increased risk of anaphylaxis following transfusions as the donated blood will
harbour IgA. T F T F T

97-Nitric oxide:

inhibits the aggregation of platelets


is the active moiety of glycerol trinitrate
production is inhibited by steroids
dilates the arterial circulation
increases pulmonary vascular resistance

Nitric oxide (NO) can be derived from endothelial cells, neuronal cells and
macrophages. It inhibits the aggregation of white cells and platelets. Steroids
inhibit macrophage nitric oxide synthases and reduce NO production. The main
source of NO is the vascular endothelium, which is continuously dilated by basal
synthesis of NO. Pulmonary vascular resistance is reduced by NO, and hence the
gas may be used in patients who are difficult to ventilate. It is the active moiety of
nitro-vasodilators, including glyceryl trinitrate. T T T T F

98-Lyme disease:

may present with a larva migrans rash


is best treated with cotrimoxazole in children
65
is caused by a spirochaete
causes cerebellar ataxia
causes Guillain–Barré syndrome

Lyme disease is caused by the spirochaete Borrelia burgdorferi, and is transmitted


by ticks carried by wild deer and other animals. The characteristic rash is called
erythema migrans, and evolves around a central bite up to 21 days after exposure.
Most cases respond to a 3-week course of Amoxil or erythromycin. Complications
include arthritis, cardiac involvement, and hepatitis. Neurological complications
include cranial nerve palsies, Guillain–Barré syndrome, cerebellar ataxia and
aseptic meningitis. Diagnosis is usually based on history and clinical findings, with
supportive serology for Borrelia. F F T T T

99-Erythema nodosum is associated with:

sarcoidosis
Bartonella henselae infection
Crohn’s disease
ulcerative colitis
streptococcal infection

Erythema nodosum describes red or dark raised ovoid lesions of 1–3 cm on the
shin, usually in girls over the age of 6 years. Systemic causes include sarcoidosis,
Crohn’s disease, ulcerative colitis and vasculitis. Infective causes include
Streptococcus infection, mycoplasma, tuberculosis, cat scratch disease (Bartonella
henselae), Epstein–Barr virus, and histoplasmosis. Sulfonamides and the
contraceptive pill can also cause this reaction. 4T

100-The following are absolute contraindications to


measles–mumps–rubella ( MMR) vaccination:

autism
attention deficit disorder
HIV infection
previous mumps infection
hepatitis B infection

66
MMR is a live vaccine, and is contraindicated in patients with significant
immunodeficiency, including those who have been on steroids during the previous
3 months, or those undergoing chemotherapy during the previous 6 months. The
vaccine should be delayed if the child is febrile, or has another live vaccination in
the previous 3 weeks. It should not be given during pregnancy. In the UK , it is
recommended that children with HIV should not receive BCG, or vaccines for
yellow fever or oral typhoid. However, they should be given routine inactivated
vaccines and inactivated polio immunisation. MMR may be given as long as the
child is not severely immunosuppressed at the time. Autism and attention deficit
disorders are not contraindications to MMR , nor is there any evidence that MMR
causes autism. 4 F

101-An 18-month-old male presents with recurrent pneumonia, ear infections


and tonsillitis. The family history includes a previous male infant death at 2
years of age from pneumococcal meningitis. The parents are therefore very
concerned and wish further investigations to be carried out.

After the results have returned, it is found that the child has absent or
decreased levels of immunoglobulins A, G and M. He has reduced responses
to blood group antibodies to immunisations and an increased percentage of E
rosettes with red blood cells. His PHA and nitroblue tetrazolium (NBT) tests
are normal. The ultimate disorder is X-linked agammaglobulinaemia.

Which of the following are appropriate treatments (more than one answer
may be given)?

A course of benzylpenicillin for an acute pneumonia and tonsillitis


Azithromycin for 3 days every fortnight as a prophylactic antibiotic
Intravenous immunoglobulin
A booster vaccine to DPT, Hib and meningitis C
Bone marrow transplantation

X-linked agammaglobulinaemia is the most common immunoglobulin deficiency


causing a deficiency in all the immunoglobulin classes. The gene is localised to the
long arm of the X chromosome, with the abnormality being in the gene for the B-
cell tyrosine kinase. It is more common in males. This disorder is associated with
an increased risk of blood malignancies. It presents after 3–6 months of age
when the level of IgG resulting from in utero placental transfer has decreased. The

67
child presents with recurrent bacterial infections in the first 2 years of life, namely
lung and sinus infections. Causes of hypogammaglobulinaemia include:

1. Common variable immune deficiency


2. Hypogammaglobulinaemia with raised IgM (X linked)
3. Transient immune deficiency of infancy.

Prenatal diagnosis is possible. Investigations show a profound failure of Bcell


development so plasma cells from the bone marrow are absent. There is an
abnormal B-cell function and number so there are isohaemagglutinins or surface
immunoglobulins. Investigations undertaken include those stated, including finding
an increase in OKT3 (49–63% of cases). Neutrophil, lymphocyte, PHA,
nitroblue tetrazolium (NBT), sweat test and T-cell function are all normal. The
PHA test measures T-cell proliferation. The T-cell lymphocytes are identified by
their ability to form rosettes with sheep red blood cells. T-cell proliferation gives
the total percentage of lymphocytes; the number is increased if no B-cells are
present. Monoclonal antibodies are now used to identify and characterize T-cells.
Monoclonal antibodies against T3 receptors measure the total Tcell number, T4
receptors, the T helper cell population and T8 receptors in the T
suppressor/cytotoxic (Ts/c) group. Treatment includes prophylactic antibiotics and
intravenous immunoglobulins. Bone marrow transplantation is carried out if
medical interventions fail. F T T T T

102-In Wiskott–Aldrich syndrome:

there are reduced numbers of large platelets


psoriasis is common
inheritance is X-linked
immune cells have defective motility
vasculitis is common
Wiskott–Aldrich syndrome (WAS) is X-linked. There is classically a triad of
thromobocytopenia, eczema, and immunodeficiency. Platelets are small and have
reduced volume. Associated features include vasculitis and autoimmunity, and a
predisposition to lymphoproliferative diseases in later life. The underlying defect is
due to loss of the WAS protein, which is important for cytoskeletal organisation
and cell motility. The disease can be cured by bone marrow transplantation, and
this should ideally be undertaken in children aged less than 5 years if a fully
matched donor is available. F F T T T

68
103-In haematopoietic stem cell transplantation:

umbilical cord blood may be used


CD34 identifies stem cells
the donor and recipient must have identical major histocompatibility complex (
MHC) antigens
donor T cells cause graft versus host disease
growth retardation is a late complication

Haematopoietic stem cells ( HSC ) may be derived from bone marrow harvests or
leukopheresis after a course of granulocyte-colony stimulating factor (GCSF).
Umbilical cord blood collected at the time of delivery is a rich source of HSC and
cord blood banks are being established. CD34 is a cell surface molecule that is
routinely used to identify and select HSC. Patients may have a fully MHC-matched
sibling donor, or there may be an unrelated matched volunteer donor. The risk of
graft versus host disease (GVHD) is greatly increased if mismatched donors are
used, although on occasion parental haplo-identical donors have to be used. GVHD
is mediated by donor T cells, and T cells can be removed from the graft to reduce
the risk. Late complications of HSCT include impaired immune reconstitution,
growth retardation, autoimmunity and endocrine dysfunction. T T F T T

104-The polymerase chain reaction (PCR):

requires monoclonal antibodies


requires forward and reverse primers
requires restriction digestion enzymes
is used to quantify human immunodeficiency virus (HIV) load
may be used to amplify RNA

In polymerase chain reaction ( PCR ) reactions, two oligonucleotide primers


(15–30 base pairs in size) are mixed with the target DNA , a mixture of
deoxyribonucleoside triphosphates (dNTPs) and thermostable Taq polymerase.
Primers are designated forward and reverse, depending on which strand of DNA
they bind. There are three stages to each PCR cycle:

 Heating to 95°C causes the DNA strands to separate (denaturation)


 Cooling to 54°C allows the primers to anneal to each strand

69
 Heating to 72°C results in incorporation of dNTPs.

The process results in exponential expansion of DNA after each cycle. Target RNA
may be amplified by using reverse transcriptase (RT) to convert it to template
DNA before use in a PCR reaction. HIV is an RNA virus that can be detected and
quantified using an RT- PCR reaction. This is now routinely used to follow the
efficacy of anti-retroviral treatment on HIV load. F T F T T

105-Kawasaki disease is associated with:

hydrops of the gallbladder


myocarditis
erythema of BCG vaccination site
desquamation of the perineum
cranial nerve palsy

The established criteria for the diagnosis of Kawasaki disease are 5 days of fever
plus four of the following:

 cervical lymphadenopathy
 non suppurative conjunctivitis
 exanthem
 swelling or desquamation of the hands or feet
 oropharyngeal inflammation.

Additional features include irritability, meningitis, iridocyclitis and arthritis.


Dilatation or hydrops of the gallbladder is recognised. Erythema of a BCG
immunisation site is a characteristic feature. Peripheral and cranial nerve palsies
may occur. The development of coronary arteritis may lead to aneurysms and
thrombosis, though all parts of the heart may be involved. 4T

106-Members of the innate immune system include:

T lymphocytes
antimicrobial peptides
insulin-like growth factors
complement C3
lectins

70
The innate or non-specific immune system is the first line of defence against
pathogens. It has many components, including mechanical barriers (skin, mucous
secretions) and soluble factors (complement, mannose-binding lectin) as well as
neutrophils and macrophages. Anti-microbial peptides, such as β-defensins are
proteins secreted by endothelial cells. Specific immunity is provided by T cells and
B cells, and results in the generation of immune memory, resulting in rapid
responses on subsequent challenge by a given pathogen. F T F T F

107-Apoptosis:

causes cellular shrinkage


is an inflammatory process
is mediated by caspases
is induced by tumour necrosis factor (TNF)
is induced by insulin

Apoptosis results in programmed cell death and is non-inflammatory. Apoptosis is


needed for embryonic development in utero and tissue homeostasis in adult life.
Apoptosis may be induced by soluble factors such as TNF-binding death receptors
or the withdrawal of growth factors (e.g. insulin-like growth factors). During
apoptosis cells shrink, chromatin condenses, with endonuclease activation and
DNA cleavage. Caspases are important mediators of apoptotic signals within cells.
Mutations in some genes or over-expression of others (e.g. bcl-2) may lead to a
failure of apoptosis and the formation of tumourogenic cell populations. T F T T F

108-Disease-associated prions:

replicate in red blood cells


resist proteinases
may be detected by polymerase chain reaction (PCR)
are inherited with mitochondria
accumulate in the tonsils
According to the prion hypothesis, disease-causing prions are corrupted versions of
naturally occurring prion proteins found on cell surfaces. Once corrupted they
resist proteinase degradation, are heat stable, and are insensitive to ultraviolet light
damage. Prions cause transmissible spongiform encephalopathies (TSEs) such as
71
BSE in cattle and Creutzfeldt–Jacob disease (CJD) in man. In 1996, new-variant
CJD was first described and is thought to be caused by bovine prions entering the
human food chain. Such prions are detectable in tonsil tissue. They do not encode
genetic material and thus are not detectable by PCR. F T F F T

109-Following delivery, infants born to HIV-infected mothers should:

receive high-dose cotrimoxazole


commence triple therapy
avoid breast feeding
be immunised with BCG
receive high-dose immunoglobulin

In the UK , the current guidelines recommend treatment of infants of HIV-positive


mothers with zidovudine (AZT) from birth. Nelfinavir is an alternative agent that
may be used, in particular if the mother is known to have developed resistance to
AZT. Triple therapy is not usually indicated at birth, but may be initiated if there
were a rising viral load or a falling CD4 count. PCR for HIV is usually performed
within 48 hours, and repeated at 6 weeks. If both are negative, therapy can be
stopped and the baby followed up until antibody negative. Babies should not be
breast fed, and should not receive BCG vaccination. Cotrimoxazole would usually
be given from 6 weeks of age. There is no indication for immunoglobulin therapy.
FFTFF

110-Recognised causes of thrombocytopenia are:

phenytoin
carbamazepine
heparin
protamine sulphate
tranexamic acid

Drug-induced thrombocytopenia may be as a result of direct toxicity on the


megakaryocytes or as a result of immune-mediated platelet consumption. Common
causes in children include phenytoin, carbamazepine, chloramphenicol, and co-
trimoxazole. Heparin may induce immune-mediated platelet elimination 7–10

72
days after starting treatment. Protamine sulphate is used to reverse heparin
anticoagulation effects and tranexamic acid is a pro-coagulant. T T T F F

111-Meningitis is suspected in a 3-week-old baby. Which of the following


treatment combinations is MOST appropriate?

Vancomycin and cefotaxime


Cefotaxime and aciclovir
Ampicillin and cefotaxime
Benzyl penicillin and gentamicin
Vancomycin and gentamicin

Under the age of 4 weeks, the common causes of meningitis include: group B
streptococci; Escherichia coli; Listeria monocytogenes. Other pathogens such as
Haemophilus influenzae, Staphylococcus, and Klebsiella spp are less frequent.
From the list of answers, the most appropriate presumptive therapy would be with
cefotaxime and ampicillin. Cefotaxime provides broad spectrum cover against
streptococci and Gram-negative organisms and the inclusion of ampicillin is
essential to cover Listeria infections. 3

112-The MOST appropriate treatment for suspected scabies infestation in a 3-


year-old child would be which of the following?

Benzyl benzoate
Ketoconazole
Permethrin
Hydrocortisone
Chlorhexidine

The immune response to the mite Sarcoptes scabiei humanis causes itchy skin
eruptions, often with an eczematous rash and excoriations. There may be burrows
on the palms, soles and digits. The mite can survive for up to 36 hours away from
the human host. All family members should be treated, and clothing and bed-linen
should be washed. Treatment should be with permethrin (Lyclear) or malathion
(Derbac) lotions. Older preparations, such as benzyl benzoate, should be avoided
in children. Ketoconazole is an antifungal agent and is not indicated. There may be
an indication for systemic antibiotics if lesions become infected. 3

73
113-Osteomyelitis of the femur is suspected in a 4-year-old child. From the list
below, which is the MOST likely organism to be involved?

Group B streptococcus
Salmonella
Mycoplasma pneumoniae
Staphylococcus aureus
Haemophilus influenzae B

Osteomyelitis 3–10 years is more common in boys than in girls and there may be
a history of trauma. The most commonly involved organism is Staphylococcus
aureus and treatment should be intravenous flucloxacillin (plus fusidic acid). Other
causes are Streptococcus pneumoniae or possibly Escherichia coli. In infants under
2 years of age alternative organisms include Group B streptococcus and
Haemophilus influenzae type B. In patients with sickle cell disease, Salmonella and
Gram-negative infections should be considered. Treatment may require surgical
debridement and antibiotics should be given for 6 weeks. 4

114-Theme: Viral infections

A Herpes simplex virus


B Human herpes virus 6
C Influenza virus
D Adenovirus
E Epstein–Barr virus
F Cytomegalovirus
G Hepatitis A
H Hepatitis B
I Measles
J Mumps
K Rubella

Scenario 1 A 15-month-old infant presents to the emergency department with


a 3-day history of high fevers without any localising signs. She suffers a short
self-limiting febrile convulsion and is admitted for observation. The next day
the fever lapses, but a red macular–papular rash develops over her trunk
and abdomen.

74
Human herpes virus 6 Correct answer

HHV-6 is thought to infect 90% of infants by 2 years of age. The pattern of high
fevers for 3 days without an obvious cause is characteristic. Febrile seizures are a
relatively common mode of presentation. The roseola rash often appears as fever
subsides, and may be macular or macular–papular and usually begins centrally,
spreading later to the limbs and resolving in 48–72 hours. Although other viruses
such as measles may cause a similar rash, the timing and fever pattern favour
HHV-6.

Scenario 2 A 10-year-old boy complains of headache and goes to bed early.


The next morning he is found to be drowsy and confused. He is admitted to
hospital where CT imaging reveals bilateral temporal lobe enhancement.

Herpes simplex virus Correct answer

Though most of the viruses listed may cause meningoencephalitis, temporal lobe
involvement is characteristic of herpes simplex virus infection. Cerebrospinal fluid
(CSF) examination usually reveals a raised cell count, often with red cells present,
and raised protein, but normal glucose level. The presence of the virus may be
confirmed using PCR detection techniques. Treatment would be with high-dose
intravenous aciclovir for 21 days.

Scenario 3 A 42-year-old man develops mild jaundice 1 month after a trip to


Thailand. His 4-year-old son now presents with vomiting and diarrhoea. The
child has been febrile and is thus receiving regular doses of paracetamol. His
liver function tests are slightly deranged.

Hepatitis A Correct answer

In early childhood hepatitis A usually causes self-limiting diarrhoea and vomiting.


Mild hepatitis is common, though children rarely develop jaundice. There is often
a nursery or home contact who has passed on the virus through the faecal–oral
route. In this case it seems to have been the father. Though hepatitis B is prevalent
in the far east, it is only transmitted by the blood borne route and does not cause
diarrhoea.

75
115-A 4-year-old, fair-skinned red-haired boy presents to the emergency
department with a long history of chronic viral illnesses associated with
diarrhoea. He appears atopic with rhinorrhoea and eczema. The GP has given
him four courses of antibiotics for presumed chronic otitis media. On
examination, the boy is small, has a fever of 38.4°C and appears pale with
small bruises over his shins. He has marked eczema over the extensor surfaces
and behind the ears but no associated superinfection. He has conjunctivitis
with red conjunctivae and thrush. Cardiovascular, respiratory and abdominal
examination is unremarkable. The bloody diarrhoea grows Campylobacter.

Investigations reveal a white cell count of 5.5x109 cells/l, a haemoglobin of 10.4


g/dl and a platelet count of 30x109 cells/l, and he is lymphopenic. An
immunoglobulin profile reveals a decreased level of IgM. The boy is
commenced on oral erythromycin.

What is the most likely diagnosis?

Acute leukaemia
Hyper-IgM syndrome
Wiskott–Aldrich syndrome
Ataxia telangiectasia
X-linked agammaglobulinaemia

Infections found and type of associated immune deficiency

T-cell deficiency B-cell deficiency Neutrophil defect


Wiscott–Aldrich syndrome Bruton’s disease Chronic granulomatous disease
Ataxia telangiectasia

Primary immunodeficiencies are:

 X-linked agammaglobulinaemia
 Hyper-IgG syndrome
 Severe combined immunodeficiency syndrome
 Wiskott–Aldrich syndrome
 Ataxia telangiectasia
 DiGeorge syndrome
 Hereditary angioneurotic oedema.
76
Antibody/B cells and Bacteria

T cells:

 Viruses, especially herpes simplex, cytomegalovirus (CMV), Epstein–Barr


and herpes zoster
 Mycobacteria, especially live in macrophages Candida

Complement (rare):

 Impaired C5–C9 complex of the complement cascade


 Bacteria (meningococcal)
 Lytic terminal complement pathway

Phagocytes:

 Bacteria, especially in those immuno-compromised or undergoing cancer


chemotherapy
 Fungi 3

116-The MMR vaccine:

Is contraindicated in patients allergic to neomycin


Should not be given within 3 weeks of another live vaccine (except oral polio
vaccine [Sabin])
Is safe in pregnancy
Commonly results in a rash with or without fever from day 5 to day 10, lasting
approximately 2 days
Is contraindicated in patients who have received an injection of immunoglobulin
within 3 months

The MMR vaccine is contraindicated in patients who are allergic to neomycin or


kanamycin. In children who have had a previous anaphylactic reaction to egg,
immunisation is not absolutely contraindicated and should be discussed with a
local paediatrician or immunisation coordinator. The MMR vaccine should not be
given within 3 weeks of another live vaccine, because this results in a suboptimal
response. Likewise, it is contraindicated in patients who have received an injection
of immunoglobulin within 3 months, because no response will be mounted in the
presence of immunoglobulin that may contain antibodies to measles, mumps or

77
rubella. Pregnancy should be avoided for at least 1 month after immunisation,
which may well result in a rash with or without fever from about day 5–10
lasting about 2 days. It is therefore sensible to provide advice on temperature
control at the time of immunisation. T T F T T

117-Theme: Immunisations

A Acellular pertussis vaccine

B Conjugate pneumococcal vaccine

C Hepatitis A vaccine

D Hepatitis B vaccine

E Influenza vaccine

F Palivizumab (anti-RSV [respiratory syncytial virus] immunoglobulin)

G Ribavirin

H Salk polio vaccine

I Single measles vaccine

J Unconjugated pneumococcal vaccine

For the following situations, choose one vaccine from the list above that should be
used instead of, or in addition to, the routine scheduled vaccines. Each item may
be used once, more than once or not at all.

Scenario 1 An ex-premature baby is discharged in October with home oxygen.


He has had the first three sets of the universal schedule (while an inpatient).

Palivizumab (anti-RSV [respiratory syncytial virus] immunoglobulin) Correct


answer

Ex-pre-term infants with chronic lung disease are at a much higher risk from
respiratory infections. RSV is a major concern for babies with chronic lung
disease, leading to high rates of hospital admission, PICU admission and even
mortality. The evidence for passive immunisation against RSV shows that it will

78
not prevent infection or hospital admission with RSV bronchiolitis (in babies with
chronic lung disease). However, it seems to reduce the severity of the illness, such
that it reduces the need for PICU admission (NNT = 10 to prevent 1 PICU
admission). There is some debate about whether this is cost-effective. Most
neonatal ICUs will arrange the immunisation of babies with chronic lung disease.

Scenario 2 A 4-year-old whose sibling is receiving chemotherapy for AML.

Influenza vaccine Correct answer

A child receiving chemotherapy for AML will be significantly immunosuppressed.


If such a child has siblings, it is important that they are immunised against
influenza, which can be devastating to an immunocompromised child. Previously a
sibling would have received inactivated polio rather than live oral polio vaccine;
however, since the change to the universal schedule (where all polio is inactivated)
this is no longer a consideration.

Scenario 3 A 3-year-old infant has a splenectomy following a road traffic


accident.

Unconjugated pneumococcal vaccine Correct answer

This child may or may not have been part of the conjugated pneumococcal vaccine
catch-up programme; however, as she is over 2 years old, it is important that she
receive the unconjugated vaccine. At this age she will be able to make a good
antibody response to the vaccine and the unconjugated vaccine protects against 30-
40 pneumococcal serotypes (compared with 7 serotypes for the conjugated
vaccine).

79

You might also like